0% found this document useful (0 votes)
15 views

Split Harrison 19

The document contains a series of questions related to rheumatology and immunology, focusing on various aspects of the immune system, autoimmune diseases, and specific conditions such as systemic lupus erythematosus and rheumatoid arthritis. Each question presents a clinical scenario or a theoretical inquiry, requiring the selection of the best response from multiple choices. The content is aimed at assessing knowledge in the field of rheumatology and immunology for educational or examination purposes.

Uploaded by

akramsattar5
Copyright
© © All Rights Reserved
We take content rights seriously. If you suspect this is your content, claim it here.
Available Formats
Download as PDF, TXT or read online on Scribd
0% found this document useful (0 votes)
15 views

Split Harrison 19

The document contains a series of questions related to rheumatology and immunology, focusing on various aspects of the immune system, autoimmune diseases, and specific conditions such as systemic lupus erythematosus and rheumatoid arthritis. Each question presents a clinical scenario or a theoretical inquiry, requiring the selection of the best response from multiple choices. The content is aimed at assessing knowledge in the field of rheumatology and immunology for educational or examination purposes.

Uploaded by

akramsattar5
Copyright
© © All Rights Reserved
We take content rights seriously. If you suspect this is your content, claim it here.
Available Formats
Download as PDF, TXT or read online on Scribd
You are on page 1/ 50

SECTION IX

Rheumatology and Immunology

QUESTIONS

DIRECTIONS: Choose the one best response to each question.

IX-1. All of the following are key features of the innate immune system EXCEPT:
A. Exclusively a feature of vertebrate animals
B. Important cells include macrophages and natural killer lymphocytes
C. Nonrecognition of benign foreign molecules or microbes
D. Recognition by germline-encoded host molecules
E. Recognition of key microbe virulence factors but no recognition of self-molecules

IX-2. A 29-year-old man with episodic abdominal pain and stress-induced edema of the lips, tongue, and
occasionally larynx is likely to have low functional or absolute levels of which of the following
proteins?
A. C1 esterase inhibitor
B. C5A (complement cascade)
C. Cyclooxygenase
D. Immunoglobulin (Ig) E
E. T-cell receptor, α chain

IX-3. Which of the following statements best describes the function of proteins encoded by the human
major histocompatibility complex (MHC) I and II genes?
A. Activation of the complement system
B. Binding to cell surface receptors on granulocytes and macrophages to initiate phagocytosis
C. Nonspecific binding of antigen for presentation to T cells
D. Specific antigen binding in response to B-cell activation to promote neutralization and
precipitation

IX-4. All of the following statements regarding primary immunodeficiency disorders are true EXCEPT:
A. Infections of the upper or lower respiratory tract suggest a defective antibody response.
B. Most are diagnosed by the presence of recurrent or unusually severe infections.
C. Recurrent infections due to Candida species suggest impaired T-cell immunity.
D. They are typically genetic diseases with Mendelian inheritance.
E. While most aspects of the immune system may be involved, innate immunity is not affected by
these disorders.

IX-5. A 19-year-old college freshman comes to the university clinic complaining of tender, painful skin
lesions in his axilla. (See Figure IX-5.) He reports that he has had similar episodes throughout his life
for which he receives antibiotics. He has a lab printout from his last episode that reports a positive
culture for Serratia marcescens. All of the following statements regarding this patient and his likely
diagnosis are true EXCEPT:
FIGURE IX-5 From Wolff W, Johnson RA, Saavedra AP: Fitzpatrick’s Color Atlas & Synopsis of Clinical
Dermatology, 7th ed. New York, NY: McGraw-Hill, 2013, Fig. 31-25.

A. Human stem cell transplantation is curative.


B. Infections with catalase-negative organisms are typical.
C. Prophylactic use of trimethoprim/sulfamethoxazole is effective in reducing risk of bacterial
infections.
D. The disease is caused by defective production of reactive oxygen species in phagolysosomes.
E. The disease is most likely transmitted by X-linked recessive inheritance.

IX-6. A 37-year-old man has recently been diagnosed with systemic hypertension. He is prescribed
lisinopril as initial monotherapy. He takes this medication as prescribed for 3 days and, on the third
day, notes that his right hand is swollen, mildly itching, and tingling. Later that evening, his lips
become swollen and he has difficulty breathing. Which of the following statements accurately
describes this condition?
A. His symptoms are due to direct activation of mast cells by lisinopril.
B. His symptoms are due to impaired bradykinin degradation by lisinopril.
C. His symptoms are unlikely to recur if he is switched to enalapril.
D. Peripheral blood analysis will show deficiency of C1 inhibitor.
E. Plasma IgE levels are likely to be elevated.

IX-7. A 28-year-old woman seeks evaluation from her primary care doctor for recurrent episodes of hives
and states that she is “allergic to cold weather.” She reports that for more than 10 years she would
develop areas of hives when exposed to cold temperatures, usually on her arms and legs. She has
never sought evaluation previously and states that, over the past several years, the occurrence of the
hives has become more frequent. Other than cold exposure, she can identify no other triggers for
development of hives. She has no history of asthma or atopy. She denies food intolerance. Her only
medication is oral contraceptive pills, which she has taken for 5 years. She lives in a single-family
home that was built 2 years ago. On examination, she develops a linear wheal after being stroked
along her forearm with a tongue depressor. Upon placing her hand in cold water, her hand becomes
red and swollen. In addition, there are several areas with a wheal and flare reaction on the arm above
the area of cold exposure. What is the next step in the management of this patient?
A. Assess for the presence of antithyroglobulin and antimicrosomal antibodies.
B. Check C1 inhibitor levels.
C. Discontinue the oral contraceptive pills.
D. Treat with cetirizine 10 mg daily.
E. Treat with cyproheptadine 8 mg daily.

IX-8. A 23-year-old woman seeks evaluation for seasonal rhinitis. She reports that she develops
symptoms yearly in the spring and fall. During this time, she develops rhinitis with postnasal drip and
cough that disrupts her sleep. In addition, she will also note itchy and watery eyes. When the
symptoms occur, she takes nonprescription loratadine, 10 mg daily, with significant improvement in
her symptoms. What is the most likely allergen(s) causing this patient’s symptoms?
A. Grass
B. Ragweed
C. Trees
D. A and B
E. B and C
F. All of the above
IX-9. You are working in the emergency department when a 3-year-old boy arrives by ambulance. He
was eating tonight when he suddenly starting wheezing, coughing, and then became progressively
less responsive. His parents are certain he did not aspirate. On arrival, his blood pressure is low, and
he is working hard to breath. You auscultate a tight wheeze bilaterally. You accurately diagnose him
with anaphylaxis and initiate appropriate therapy. Which of the following is true regarding
anaphylaxis?
A. An atopic history is a risk factor for anaphylaxis to penicillin therapy.
B. Anaphylaxis most often onsets 1–2 hours after antigen exposure.
C. Older age is associated with improved outcomes in anaphylaxis.
D. The failure to use epinephrine within the first 20 minutes of symptoms is a risk factor for death
due to anaphylaxis.
E. Intravenous glucocorticoids are effective for acute anaphylaxis.

IX-10. Rheumatic fever develops due to an autoimmune process. Which of the following mechanisms of
autoimmunity is primarily responsible for the development of rheumatic fever?
A. Endocrine abnormalities
B. Increased B-cell function
C. Intrinsic cytokine imbalance
D. Increased T-cell help due to cytokine stimulation
E. Molecular mimicry

IX-11. Which of the following describes the pathophysiologic autoimmune mechanism responsible for
Graves disease?
A. Antibody-dependent cellular cytotoxicity
B. Complement-activating autoantibody
C. Inactivating autoantibody
D. Stimulating autoantibody
E. T-cell–mediated cellular cytotoxicity

IX-12. Which of the following autoantibodies is least likely to be present in a patient with systemic lupus
erythematosus?
A. Anti-dsDNA
B. Antinuclear antibodies
C. Anti-La (SS-B)
D. Antiphospholipid
E. Antierythrocyte

IX-13. A 23-year-old woman is evaluated by her primary care physician because she is concerned that
she may have systemic lupus erythematosus after hearing a public health announcement on the radio.
She has no significant past medical history, and her only medication is occasional ibuprofen. She is
not sexually active and works in a grocery store. She reports that she has had intermittent oral ulcers
and right knee pain. Physical examination shows no evidence of alopecia, skin rash, or joint
swelling/inflammation. Her blood work shows that she has a positive antinuclear antibody (ANA) at
a titer of 1:40, but no other abnormalities. Which of the following statements is true?
A. Four diagnostic criteria are required to be diagnosed with systemic lupus erythematosus; this
patient has three.
B. Four diagnostic criteria are required to be diagnosed with systemic lupus erythematosus; this
patient has two.
C. If a urinalysis shows proteinuria, she will meet criteria for systemic lupus erythematosus.
D. She meets criteria for systemic lupus erythematosus because she has three criteria for disease.
E. The demonstration of a positive ANA alone is adequate to diagnose systemic lupus
erythematosus.
IX-14. A 32-year-old woman with long-standing diagnosis of systemic lupus erythematosus is evaluated
by her rheumatologist as routine follow-up. A new cardiac murmur is heard, and an echocardiogram
is ordered. She is feeling well and has no fevers, weight loss, or preexisting cardiac disease. A
vegetation on the mitral valve is demonstrated. Which of the following statements is true?
A. Blood cultures are unlikely to be positive.
B. Glucocorticoid therapy has been proven to lead to improvement in this condition.
C. Pericarditis is frequently present concomitantly.
D. The lesion has a low risk of embolization.
E. The patient has been surreptitiously using injection drugs.

IX-15. A 24-year-old woman is newly diagnosed with systemic lupus erythematosus. Which of the
following organ system complications is she most likely to have over the course of her lifetime?
A. Cardiopulmonary
B. Cutaneous
C. Hematologic
D. Musculoskeletal
E. Renal

IX-16. A 45-year-old African American woman with systemic lupus erythematosus (SLE) presents to the
emergency department with complaints of headache and fatigue. Her prior manifestations of SLE
have been arthralgias, hemolytic anemia, malar rash, and mouth ulcers, and she is known to have
high titers of antibodies to double-stranded DNA. She currently is taking prednisone, 5 mg daily, and
hydroxychloroquine, 200 mg daily. On presentation, she is found to have a blood pressure of
190/110 mmHg with a heart rate of 98 bpm. A urinalysis shows 25 red blood cells (RBCs) per high-
power field with 2+ proteinuria. No RBC casts are identified. Her blood urea nitrogen is 88 mg/dL,
and creatinine is 2.6 mg/dL (baseline 0.8 mg/dL). She has not previously had renal disease related to
SLE and is not taking nonsteroidal anti-inflammatory drugs. She denies any recent illness, decreased
oral intake, or diarrhea. What is the most appropriate next step in the management of this patient?

A. Initiate cyclophosphamide, 500 mg/m2 body surface area intravenously (IV), and plan to repeat
monthly for 3–6 months.
B. Initiate hemodialysis.
C. Initiate high-dose steroid therapy (IV methylprednisolone, 1000 mg daily for 3 doses, followed by
oral prednisone, 1 mg/kg daily) and mycophenolate mofetil, 2 g daily.
D. Initiate plasmapheresis.
E. Withhold all therapy until renal biopsy is performed.

IX-17. A 27-year-old woman is admitted to the intensive care unit after recent delivery of a full-term
infant 3 days prior. The patient was found to have right hemiparesis and a blue left hand. Physical
examination is also notable for livedo reticularis. Her laboratories were notable for a white blood cell
(WBC) count of 10.2/μL, hematocrit of 35%, and platelet count of 13,000/μL. Her blood urea
nitrogen (BUN) is 36 mg/dL, and her creatinine is 2.3 mg/dL. Although this pregnancy was
uneventful, the three prior pregnancies resulted in early losses. A peripheral smear shows no evidence
of schistocytes. Which of the following laboratory studies will best confirm the underlying etiology of
her presentation?
A. Anticardiolipin antibody panel
B. Antinuclear antibody
C. Doppler examination of her left arm arterial tree
D. Echocardiography
E. Magnetic resonance imaging (MRI) of her brain

IX-18. A 28-year-old woman comes to the emergency department complaining of 1 day of worsening
right leg pain and swelling. She drove in a car 8 hours back from a hiking trip 2 days ago and then
noticed some pain in the leg. At first she thought it was due to exertion, but it has worsened over the
day. Her only past medical history is related to difficulty getting pregnant, with two prior
spontaneous abortions. Her physical examination is notable for normal vital signs, heart, and lung
examination. Her right leg is swollen from the mid-thigh down and is tender. Doppler studies
demonstrate a large deep venous thrombosis in the femoral and ileac veins extending into the pelvis.
Laboratory studies on admission prior to therapy show normal electrolytes, normal WBC and platelet
counts, normal prothrombin time, and an activated partial thromboplastin time 3× normal. Her
pregnancy test is negative. Low-molecular-weight heparin therapy is initiated in the emergency
department. Subsequent therapy should include which of the following?
A. Rituximab 375 mg/m2 per week for 4 weeks
B. Warfarin with international normalized ratio (INR) goal of 2.0–3.0 for 3 months
C. Warfarin with INR goal of 2.0–3.0 for 12 months
D. Warfarin with INR goal of 2.5–3.5 for life
E. Warfarin with an INR goal of 2.5–3.5 for 12 months followed by daily aspirin for life

IX-19. Patients with antiphospholipid syndrome will often falsely test positive for which of the following
infectious diseases?
A. Malaria
B. Human immunodeficiency virus (HIV)
C. Schistosomiasis
D. Hepatitis C
E. Syphilis

IX-20. Which of the following is the most frequent site of joint involvement in established rheumatoid
arthritis?
A. Distal interphalangeal joint
B. Hip
C. Knee
D. Spine
E. Wrist

IX-21. In patients with established rheumatoid arthritis, all of the following pulmonary radiographic
findings may be explained by their rheumatologic condition EXCEPT:
A. Bilateral interstitial infiltrates
B. Bronchiectasis
C. Lobar infiltrate
D. Solitary pulmonary nodule
E. Unilateral pleural effusion

IX-22. Which of the following is the earliest plain radiographic finding of rheumatoid arthritis?
A. Juxta-articular osteopenia
B. No abnormality
C. Soft tissue swelling
D. Subchondral erosions
E. Symmetric joint space loss

IX-23. All of the following are characteristic extra-articular manifestations of rheumatoid arthritis
EXCEPT:
A. Anemia
B. Cutaneous vasculitis
C. Pericarditis
D. Secondary Sjögren syndrome
E. Thrombocytopenia
IX-24. All of the following agents have been shown to have disease-modifying antirheumatic drug
(DMARD) efficacy in patients with rheumatoid arthritis EXCEPT:
A. Infliximab
B. Leflunomide
C. Methotrexate
D. Naproxen
E. Rituximab

IX-25. All of the following are characteristics of Felty syndrome EXCEPT:


A. Neutropenia
B. Nodular rheumatoid arthritis
C. Occurs in the late stages of rheumatoid arthritis
D. Splenomegaly
E. Thrombocytopenia

IX-26. Which of the following malignancies are patients with rheumatoid arthritis specifically at higher
risk for?
A. Colon cancer
B. Lung cancer
C. Lymphoma
D. Melanoma
E. Glioblastoma multiforme

IX-27. Which of the following is the most common clinical presentation of acute rheumatic fever?
A. Carditis
B. Chorea
C. Erythema marginatum
D. Polyarthritis
E. Subcutaneous nodules

IX-28. A 19-year-old recent immigrant from Ethiopia comes to your clinic to establish primary care. She
currently feels well. Her past medical history is notable for a recent admission to the hospital for
new-onset atrial fibrillation. As a child in Ethiopia, she developed an illness that caused uncontrolled
flailing of her limbs and tongue lasting approximately 1 month. She also has had three episodes of
migratory large-joint arthritis during her adolescence that resolved with pills that she received from
the pharmacy. She is currently taking metoprolol and warfarin and has no known drug allergies.
Physical examination reveals an irregularly irregular heart beat with normal blood pressure. Her
point of maximal impulse (PMI) is most prominent at the midclavicular line and is normal in size. An
early diastolic rumble and 3/6 holosystolic murmur are heard at the apex. A soft early diastolic
murmur is also heard at the left third intercostal space. You refer her to a cardiologist for evaluation
of valve replacement and echocardiography. What other intervention might you consider at this
time?
A. Daily aspirin
B. Daily doxycycline
C. Low-dose corticosteroids
D. Monthly penicillin G injections
E. Penicillin G injections as needed for all sore throats

IX-29. Most of the manifestations of acute rheumatic fever present approximately 3 weeks after the
precipitating group A streptococcal infection. Which manifestation may present several months after
the precipitating infection?
A. Chorea
B. Erythema marginatum
C. Fever
D. Polyarthritis
E. Subcutaneous nodules

IX-30. A patient with a diagnosis of scleroderma who has diffuse cutaneous involvement presents with
malignant hypertension, oliguria, edema, hemolytic anemia, and renal failure. You make a diagnosis
of scleroderma renal crisis. Which of the following is the recommended treatment?
A. Captopril
B. Carvedilol
C. Clonidine
D. Diltiazem
E. Nitroprusside

IX-31. Which of the following is nearly twice as common in patients with diffuse cutaneous systemic
sclerosis than in limited cutaneous systemic sclerosis?
A. Esophageal involvement
B. Pulmonary arterial hypertension
C. Pulmonary fibrosis
D. Raynaud phenomenon
E. Skin involvement

IX-32. Which of the following autoantibodies is typically present in high titers in patients with mixed
connective tissue disease?
A. Anti-centromere
B. Anti-La
C. Anti-Ro
D. Anti–Scl-70
E. Anti–U1-RNP

IX-33. A 57-year-old woman with depression and chronic migraine headaches reports several years of
dry mouth and dry eyes. Her primary complaint is that she can no longer eat her favorite crackers,
although she does report photosensitivity and eye burning on further questioning. She has no other
associated symptoms. Examination shows dry, erythematous, sticky oral mucosa. All of the following
tests are likely to be positive in this patient EXCEPT:
A. La/SS-B antibody
B. Ro/SS-A antibody
C. Schirmer I test
D. Scl-70 antibody
E. Sialometry

IX-34. A patient with primary Sjögren syndrome that was diagnosed 6 years ago and treated with tear
replacement for symptomatic relief notes continued parotid swelling for the last 3 months. She has
also noted enlarging posterior cervical lymph nodes. Evaluation shows leukopenia and low C4
complement levels. What is the most likely diagnosis?
A. Amyloidosis
B. Chronic pancreatitis
C. HIV infection
D. Lymphoma
E. Secondary Sjögren syndrome

IX-35. Which of the following is the most common extraglandular manifestation of primary Sjögren
syndrome?
A. Arthralgias/arthritis
B. Lymphoma
C. Peripheral neuropathy
D. Raynaud phenomenon
E. Vasculitis

IX-36. A 43-year-old Japanese woman presents to your clinic with 6 months of dry irritated eyes, dry
mouth, and cheek swelling. On examination, she has parotic gland enlargement bilaterally.
Sialometry is abnormal. Biopsy of the minor salivary glands in the lip shows granulomatous
inflammation. Serologies show a negative SS-A and SS-B antibodies. Which of the following is the
most likely diagnosis?
A. Systemic sclerosis
B. Sarcoidosis
C. Sjögren syndrome
D. HIV-associated sicca syndrome
E. Eosinophilic granulomatosis with polyangiitis

IX-37. Histocompatibility antigen HLA-B27 is present in what percentage of North American patients
with ankylosing spondylitis?
A. 10%
B. 30%
C. 50%
D. 90%
E. 100%

IX-38. Which of the following is the most common extra-articular manifestation of ankylosing
spondylitis?
A. Anterior uveitis
B. Aortic insufficiency
C. Inflammatory bower disease
D. Pulmonary fibrosis
E. Third-degree heart block

IX-39. Mr. Charleston is a 25-year-old man seeing his primary care physician for evaluation of low back
pain. The pain is severe, worse in the morning, improved with exercise, and worse with rest; in
particular, nighttime sleeping is difficult. He does feel quite stiff in the morning for at least 30
minutes. An MRI of his lower back is obtained and shows active inflammation in the sacroiliac joint.
On further questioning, he reports a history of unilateral eye redness treated with corticosteroids
about 2 years ago. A test for HLA-B27 is positive. Which of the following is first-line therapy for his
condition?
A. Infliximab
B. Naproxen
C. Prednisone
D. Rituximab
E. Tramadol

IX-40. Mr. Husten is a 27-year-old man seen at his primary care physician’s office for evaluation of
painful arthritis involving the right knee associated with diffuse bilateral finger swelling. He is
otherwise healthy but does recall a severe bout of diarrheal illness about 3–4 weeks prior that
spontaneously resolved. He works as a recreation supervisor at a daycare center and said many of the
children had a similar diarrheal illness. He takes no medications and reports rare marijuana use. On
review of systems, he reports painful urination. Examination shows inflammatory arthritis of the right
knee, dactylitis, and normal genitourinary examination. He is diagnosed with reactive arthritis.
Which of the following was the most likely etiologic agent of his diarrhea?
A. Campylobacter jejuni
B. Clostridium difficile
C. Escherichia coli
D. Helicobacter pylori
E. Shigella flexneri

IX-41. Which of the following statements regarding the arthritis of Whipple disease is true?
A. Arthritis is a rare finding in Whipple disease.
B. Joint manifestations are usually concurrent with gastrointestinal symptoms and malabsorption.
C. Radiography frequently shows joint erosions.
D. Synovial fluid examination is unlikely to show polymorphonuclear cells.
E. None of the above

IX-42. Which of the following definitions best fits the term enthesitis?
A. A palpable vibratory or crackling sensation elicited with joint motion
B. Alteration of joint alignment so that articulating surfaces incompletely approximate each other
C. Inflammation at the site of tendinous or ligamentous insertion into bone
D. Inflammation of the periarticular membrane lining the joint capsule
E. Inflammation of a saclike cavity near a joint that decreases friction

IX-43. All of the following help distinguish psoriatic arthritis from other joint disorders EXCEPT:
A. Dactylitis
B. Enthesitis
C. Nail pitting
D. Presence of diarrhea
E. Shortening of digits

IX-44. Which cardiac valvular lesion is most common in patients with ankylosing spondylitis?
A. Aortic regurgitation
B. Mitral regurgitation
C. Mitral stenosis
D. Pulmonic stenosis
E. Tricuspid regurgitation

IX-45. All of the following vasculitic syndromes are thought to be due to immune complex deposition
EXCEPT:
A. Cryoglobulinemic vasculitis
B. Granulomatosis with polyangiitis
C. Henoch-Schönlein purpura
D. Polyarteritis nodosa associated with hepatitis B
E. Serum sickness

IX-46. A 40-year-old male presents to the emergency department with 2 days of low-volume hemoptysis.
He reports that he has been coughing up 2–5 tablespoons of blood each day. He does report mild
chest pain, low-grade fevers, and weight loss. In addition, he has had about 1 year of severe upper
respiratory symptoms including frequent epistaxis and purulent discharge treated with several
courses of antibiotics. Aside from mild hyperlipidemia, he is otherwise healthy. His only medications
are daily aspirin and lovastatin. On physical examination, he has normal vital signs, and upper
airway is notable for saddle nose deformity and clear lungs. A computed tomography (CT) of the
chest shows multiple cavitating nodules, and urinalysis shows red blood cells. Which of the following
tests offers the highest diagnostic yield to make the appropriate diagnosis?
A. Deep skin biopsy
B. Percutaneous kidney biopsy
C. Pulmonary angiogram
D. Surgical lung biopsy
E. Upper airway biopsy

IX-47. An 84-year-old woman is seen by her primary care physician for evaluation of severe headaches.
She noted these several weeks ago, and they have been getting worse. Although she has not had any
visual aura, she is concerned that she has been intermittently losing vision in her left eye for the last
few days. She denies new weakness or numbness, but she does report jaw pain with eating. Her past
medical history includes coronary artery disease requiring a bypass grafting 10 years prior, diabetes
mellitus, hyperlipidemia, and mild depression. Full review of symptoms is notable for night sweats
and mild low back pain particularly prominent in the morning. Which of the following is the next
most appropriate step?
A. Aspirin 975 mg orally daily
B. Measurement of erythrocyte sedimentation rate
C. Immediate initiatinon of glucocorticoid
D. Referral for temporal artery biopsy
E. Referral for ultrasound of temporal artery

IX-48. A 54-year-old man is evaluated for cutaneous vasculitis and peripheral nephropathy. Because of
concomitant renal dysfunction, he undergoes kidney biopsy that shows glomerulonephritis.
Cryoglobulins are demonstrated in the peripheral blood. Which of the following laboratory studies
should be sent to determine the etiology?
A. Hepatitis B surface antigen
B. Antineutrophil cytoplasmic antibody (ANCA)
C. Hepatitis C polymerase chain reaction
D. HIV antibody
E. Rheumatoid factor

IX-49. An 18-year-old man is admitted to the hospital with acute onset of crushing substernal chest pain
that began abruptly 30 minutes ago. He reports the pain radiating to his neck and right arm. He has
otherwise been in good health. He currently plays trumpet in his high school marching band but does
not participate regularly in aerobic activities. On physical examination, he is diaphoretic and
tachypneic. His blood pressure is 100/48 mmHg and heart rate is 110 bpm. His cardiovascular
examination has a regular rhythm but is tachycardic. A 2/6 holosystolic murmur is heard best at the
apex and radiates to the axilla. His lungs have bilateral rales at the bases. The electrocardiogram
demonstrates 4 mm of ST elevation in the anterior leads. On further questioning regarding his past
medical history, he recalls having been told that he was hospitalized for some problem with his heart
when he was 2 years old. His mother, who accompanies him, reports that he received aspirin and γ-
globulin as treatment. Since that time, he has required intermittent follow-up with echocardiography.
What is the most likely cause of this patient’s acute coronary syndrome?
A. Dissection of the aortic root and left coronary ostia
B. Presence of a myocardial bridge overlying the left anterior descending artery
C. Thrombosis of a coronary artery aneurysm
D. Vasospasm following cocaine ingestion
E. Vasculitis involving the left anterior descending artery

IX-50. You are seeing in follow-up a 46-year-old man who, 6 months ago, presented to the hospital
acutely with hemoptysis, diffuse nodular pulmonary infiltrates, and glomerulonephritis. Workup
revealed a positive serologic study for antibodies against cytoplasmic ANCA, and he was eventually
diagnosed with granulomatosis with polyangiitis. Treatment was initiated with high-dose
glucocorticoids and daily cyclophosphamide with excellent clinical response. You are ready today to
have the patient transition from induction therapy with cyclophosphamide to maintenance therapy
with azathioprine. What blood test should you check before starting azathioprine?
A. ANCA titers
B. Cryoglobulins
C. CYP3A4 genotyping
D. Glucose-6-phosphate dehydrogenase enzyme levels
E. Thiopurine methyltransferase enzyme activity

IX-51. All of the following vascular beds are typically affected by polyarteritis nodosa EXCEPT:
A. Cerebral arteries
B. Coronary arteries
C. Pulmonary arteries
D. Renal arteries
E. Splanchnic arteries

IX-52. Lung biopsy has the greatest diagnostic yield in which of the following vasculitic syndromes?
A. Cryoglobulinemic vasculitis
B. Cutaneous vasculitis
C. Granulomatosis with polyangiitis (Wegener)
D. IgA vasculitis (Henoch-Schönlein)
E. Polyarteritis nodosa

IX-53. In a patient with suspected granulomatosis with polyangiitis (Wegener), which of the following
lung radiologic findings is least likely?
A. Bronchiectasis
B. Endobronchial stenosis
C. Multiple cavitating nodules
D. Nodular infiltrates
E. Solitary cavitating nodule

IX-54. Which of the following is required for the diagnosis of Behçet disease?
A. Large-vessel vasculitis
B. Pathergy test
C. Recurrent oral ulceration
D. Recurrent genital ulceration
E. Uveitis

IX-55. A 25-year-old woman presents with a complaint of painful mouth ulcerations. She describes these
lesions as shallow ulcers that last for 1 or 2 weeks. The ulcers have been appearing for the last 6
months. For the last 2 days, the patient has had a painful red eye. She has had no genital ulcerations,
arthritis, skin rashes, or photosensitivity. On physical examination, the patient appears well
developed and in no distress. She has a temperature of 37.6°C (99.7°F), heart rate of 86 bpm, blood
pressure of 126/ 72 mmHg, and respiratory rate of 16 breaths/min. Examination of the oral mucosa
reveals two shallow ulcers with a yellow base on the buccal mucosa. The ophthalmologic
examination is consistent with anterior uveitis. The cardiopulmonary examination is normal. She has
no arthritis, but medially on the right thigh, there is a palpable cord in the saphenous vein.
Laboratory studies reveal an erythrocyte sedimentation rate of 68 seconds. WBC count is 10,230/μL
with a differential of 68% polymorphonuclear cells, 28% lymphocytes, and 4% monocytes. The
antinuclear antibody and anti-dsDNA antibody are negative. C3 is 89 mg/dL, and C4 is 24 mg/dL.
What is the most likely diagnosis?
A. Behçet syndrome
B. Bullous pemphigoid
C. Discoid lupus erythematosus
D. Sjögren syndrome
E. Systemic lupus erythematosus

IX-56. All of the following are known complications of Behçet syndrome EXCEPT:
A. Arterial thrombosis
B. Central nervous system involvement
C. Deep vein thrombosis
D. Pulmonary artery vasculitis
E. All of the above are known complications of Behçet’s syndrome.

IX-57. An elevation in which of the following serum enzymes is the most sensitive indicator of myositis?
A. Aldolase
B. Creatinine kinase
C. Glutamic-oxaloacetic transaminase
D. Glutamate pyruvate transaminase
E. Lactate dehydrogenase

IX-58. A 64-year-old woman is evaluated for weakness. She has had several weeks of difficulty brushing
her teeth and combing her hair. She has also noted a rash on her face. Examination is notable for a
heliotrope rash and proximal muscle weakness. Serum creatinine kinase (CK) is elevated, and she is
diagnosed with dermatomyositis. After evaluation by a rheumatologist, she is found to have anti-Jo-1
antibodies. She is also likely to have which of the following findings?
A. Ankylosing spondylitis
B. Inflammatory bowel disease
C. Interstitial lung disease
D. Primary biliary cirrhosis
E. Psoriasis

IX-59. A 63-year-old woman is evaluated for a rash on her eyes and fatigue for 1 month. She reports
difficulty with arm and leg strength and constant fatigue, but no fevers or sweats. She also has noted
that she has a red discoloration around her eyes. She has hypothyroidism but is otherwise well. On
examination, she has a heliotrope rash and proximal muscle weakness. A diagnosis of
dermatomyositis is made after demonstration of elevated serum creatinine kinase and confirmatory
electromyograms. Which of the following studies should be performed as well to look for associated
conditions?
A. Mammogram
B. Serum antinuclear antibody measurement
C. Stool examination for ova and parasites
D. Thyroid-stimulating immunoglobulins
E. Titers of antibodies to varicella-zoster

IX-60. You are seeing Mr. Blumenthal today, who has been your long-term patient. He has a history of
coronary artery disease, suffering a lateral myocardial infarction 1 year ago. At that time, he was
started on simvastatin, aspirin, metoprolol, and lisinopril. About 2 months ago, he started noting
thigh and shoulder soreness. One month after onset, his muscle pain had increased and he was
noticing weakness. His CK was elevated to 8× the upper limit of normal. His simvastatin was
discontinued 3 weeks ago. Today, he reports that his pain has continued and, if anything, is worse
than a month ago. His CK is 12× the upper limit of normal. What is the next best test to establish a
diagnosis?
A. Antibody against 3-hydroxy-3-methylglutaryl– coenzyme A reductase (HMGCR)
B. Antinuclear antibody (ANA)
C. Anti-Jo-1 antibody
D. Antibody against signal recognition particle (SRP)
E. Aldolase levels

IX-61. A 47-year-old man is evaluated for 1 year of recurrent episodes of bilateral ear swelling. The ear
is painful during these events, and the right ear has become floppy. He is otherwise healthy and
reports no illicit habits. He works in an office, and his only sport is tennis. On examination, the left
ear has a beefy red color and the pinna is tender and swollen; the earlobe appears minimally swollen
but is neither red nor tender. Which of the following is the most likely explanation for this finding?
A. Behçet syndrome
B. Cogan syndrome
C. Hemoglobinopathy
D. Recurrent trauma
E. Relapsing polychondritis

IX-62. All of the following have been implicated in the proposed pathogenesis of sarcoidosis EXCEPT:
A. Exposure to mold
B. Genetic susceptibility
C. Immune response to mycobacterial proteins
D. Infection with Propionibacterium acnes
E. Malignant expansion of helper T cells

IX-63. Which of the following statements regarding pulmonary sarcoidosis is true?


A. Lung involvement is the second most common manifestation of sarcoidosis, behind only
cutaneous involvement.
B. Obstructive disease is a rare manifestation of pulmonary sarcoidosis.
C. Pulmonary hypertension never responds to therapy in sarcoidosis patients.
D. Pulmonary infiltrates in sarcoidosis tend to be predominantly an upper lobe process.
E. The presence of cough should prompt evaluation for a cause other than pulmonary sarcoidosis.

IX-64. You are seeing Mr. Blanko, a 55-year-old white man with a history of sarcoidosis. He ran out of
prednisone about 2 months prior to seeing you and, except for some constipation, feels well. A
metabolic panel reveals a calcium of 12.2 mg/dL (normal up to 10.5 mg/dL). You know that
sarcoidosis can be associated with hypercalcemia. Which of the following is the correct mechanism
for sarcoidosis-associated hypercalcemia?
A. Direct granulomatous involvement of the axial skeleton causing calcium release from bones
B. Direct stimulation of increased intestinal calcium absorption
C. Increased parathyroid hormone production
D. Increased production of 1,25-dihydroxyvitamin D
E. Increased production of 25-hydroxyvitamin D

IX-65. In which population is cardiac sarcoidosis most common?


A. African American
B. East European
C. Japanese
D. South American
E. Australian

IX-66. Which of the following is a potential cardiac manifestation of sarcoidosis?


A. Dilated cardiomyopathy
B. Heart block
C. Valvular stenosis
D. Ventricular tachyarrhythmias
E. All of the above
IX-67. You are rounding on Mr. Spareti today. He is a 34-year-old man who presented to you with
unexplained pancreatitis 2 weeks ago. Imaging of his pancreas showed diffuse pancreatic
enlargement. He denies any alcohol intake and did not have any gallstones on imaging. Interestingly,
on examination, he also has marked lacrimal gland and submandibular gland enlargement. Biopsy of
his submandibular gland is pictured in Figure IX-67. The cells pictured in the figure stained strongly
positive for IgG4, CD19, and CD138. Which of the following is the appropriate therapy?

FIGURE IX-67

A. Thalidomide and dexamethasone


B. Cytomegalovirus (CMV) immunoglobulin and ganciclovir
C. Systemic chemotherapy
D. Prednisone
E. Anakinra

IX-68. You are seeing a 19-year-old woman today in consultation for recurrent fevers. She reports
several years of fevers, occurring on average every 2–3 months. These episodes are unpredictable,
although she thinks they may occur in times of psychological stress. Each febrile episode lasts 2–3
days. She also has recurrent episodes of abdominal pain. Repeated blood cultures have been negative,
even during acute febrile episodes. Similarly, abdominal CT scans have shown no obvious etiology for
her pain. During one episode, she underwent an exploratory laparotomy, which showed peritoneal
adhesions and a sterile neutrophilic peritoneal exudate. She also notes that when she exercises, she
develops intense muscle pains that last for days. An extensive serologic search for autoantibodies
returned negative, including antinuclear antibodies. Which of the following is the most likely
diagnosis?
A. Familial Mediterranean fever
B. Lymphoma
C. Relapsing fever
D. Subacute bacterial endocarditis
E. Systemic lupus erythematosus

IX-69. You are seeing a 19-year-old woman whom you just diagnosed with familial Mediterranean fever.
Which of the following medications would you prescribe to reduce attacks and help prevent the
development of systemic amyloidosis?
A. Colchicine
B. Cyclosporine
C. Diflunisal
D. Prednisone
E. Thalidomide

IX-70. Which of the following joints is often spared by osteoarthritis?


A. Cervical spine
B. Distal interphalangeal joint
C. Hip
D. Proximal interphalangeal joint
E. Wrist

IX-71. Which of the following statements regarding osteoarthritis is true?


A. During the diagnostic workup of a suspected osteoarthritic joint, MRI is warranted to evaluate for
any other causes.
B. Loss of cartilage causes pain due to direct stimulation of pain receptors in joint cartilage itself.
C. Osteoarthritis is the second most common cause of arthritis, behind rheumatoid arthritis.
D. Synovial fluid white blood cell count is usually <1000 cells/μL in osteoarthritis.
E. The severity of radiographic changes in osteoarthritis correlates well with symptoms.

IX-72. You are seeing Mrs. Hudson today, a 60-year-old obese woman with bilateral knee osteoarthritis.
Mrs. Hudson describes pain most days and limiting pain at least 2 days per week. She has tried
activity modification (walking less) without success. All of the following therapies have been shown
to be efficacious in treating osteoarthritis symptoms EXCEPT:
A. Acetaminophen
B. Glucocorticoid steroid intra-articular injections
C. Glucosamine-chondroitin
D. Naproxen
E. Total joint arthroplasty

IX-73. You are seeing Mr. Hinsley, a 72-year-old man with only a history of hypertension on
hydrochlorothiazide. He presents today with acute, excruciating knee pain. On examination, his knee
is warm, mildly erythematous, swollen, and tender to the touch or passive movement. Microscopic
examination of joint fluid is shown in Figure IX-73A. What is Mr. Hinsley’s most likely metabolic
derangement?
FIGURE IX-73A

A. Acute bacterial joint infection


B. Antibodies to antinuclear antigens
C. Hyaline cartilage degeneration
D. Increased production of inorganic pyrophosphate
E. Uric acid overproduction

IX-74. You are planning on starting allopurinol for Ms. Maggy for a new diagnosis of gouty arthritis.
Which of the following best describes appropriate dosing strategies for allopurinol?
A. Allopurinol and azathioprine are commonly used together in the treatment of gout.
B. Allopurinol dosing should be adjusted for liver function.
C. Allopurinol dosing should be titrated to achieve a serum uric acid level <6 mg/dL.
D. Allopurinol should be avoided when patients are taking colchicine.
E. Allopurinol toxicity is more common in patients expressing HLA-B27.

IX-75. A 42-year-old woman is seen in her primary care doctor’s office complaining of diffuse pains and
fatigue. She has a difficult time localizing the pain to any particular joint or location, but reports it
affects her upper and lower extremities, neck, and hips. It is described as achy and 10 out of 10 in
intensity. She feels that her joints are stiff but does not notice that it is worse in the morning. The
pain has been present for the past 6 months and is increasing in intensity. She has tried both over-
the-counter ibuprofen and acetaminophen without significant relief. The patient feels as if the pain is
interfering with her ability to get restful sleep and is making it difficult for her to concentrate. She
has missed multiple days of work as a waitress and fears that she will lose her job. There is a medical
history of depression and obesity. The patient currently is taking venlafaxine sustained release 150
mg daily. She has a family history of rheumatoid arthritis in her mother. She smokes one pack of
cigarettes daily. On physical examination, vital signs are normal. Body mass index is 36 kg/m2. Joint
examination demonstrates no erythema, swelling, or effusions. There is diffuse pain with palpation at
the insertion points of the suboccipital muscles, at the midpoint of the upper border of the trapezius
muscle, along the second costochondral junction, at the lateral epicondyles, and along the medial fat
pad of the knees. All of the following statements regarding the cause of this patient’s diffuse pain
syndrome are true EXCEPT:
A. Cognitive dysfunction, sleep disturbance, anxiety, and depression are common comorbid
neuropsychological conditions.
B. Pain in this syndrome is associated with increased evoked pain sensitivity.
C. Pain in this syndrome is often localized to specific joints.
D. This syndrome is present in 2%–5% of the general population, but increases in prevalence to 20%
or more of patients with degenerative or inflammatory rheumatic disorders.
E. Women are nine times more likely than men to be affected by this syndrome.

IX-76. A 36-year-old woman presents to your office with diffuse pain throughout her body associated
with fatigue, insomnia, and difficulty concentrating. She finds the pain difficult to localize but reports
that it is 7–8 out of 10 in intensity and not relieved by nonsteroidal anti-inflammatory medications.
She has a long-standing history of generalized anxiety disorder and is treated with sertraline 100 mg
daily as well as clonazepam 1 mg twice daily. On examination, she has pain with palpation at several
musculoskeletal sites. Her laboratory examination demonstrates a normal complete blood count, basic
metabolic panel, erythrocyte sedimentation rate, and rheumatoid factor. You diagnose her with
fibromyalgia. All of the following therapies are recommended as part of the treatment plan for
fibromyalgia EXCEPT:
A. An exercise program that includes strength training, aerobic exercise, and yoga
B. Cognitive-behavioral therapy for insomnia
C. Milnacipran
D. Oxycodone
E. Pregabalin

IX-77. A 53-year-old woman presents to your clinic complaining of fatigue and generalized pain that
have worsened over 2 years. She also describes irritability and poor sleep and is concerned that she is
depressed. She reveals that she was recently separated from her husband and has been stressed at
work. Which of the following elements in her history and physical examination would meet American
College of Rheumatology criteria for diagnosis of fibromyalgia?
A. Diffuse chronic pain and abnormal sleep
B. Diffuse pain without other etiology and evidence of major depression
C. Major depression, life stressor, chronic pain, and female sex
D. Major depression and pain on palpation at 6 of 18 tender point sites
E. Widespread chronic pain and pain on palpation at 11 of 18 tender point sites

IX-78. A 42-year-old man is found to have the following finding on a physical examination (Figure IX-
78). All of the following conditions are associated with this finding EXCEPT:
FIGURE IX-78 Reprinted from the Clinical Slide Collection on the Rheumatic Diseases, © 1991, 1995.
Used by permission of the American College of Rheumatology.

A. Chronic obstructive pulmonary disease


B. Cyanotic congenital heart disease
C. Cystic fibrosis
D. Hepatocellular carcinoma
E. Hyperthyroidism

IX-79. A 52-year-old man presented to his primary care physician complaining of new-onset pain in the
knuckles of his index and middle fingers of both hands. On examination, the second and third
metacarpophalangeal (MCP) joints of both hands are swollen and tender. The rest of his physical
examination is normal. His past medical history is only notable for hyperlipidemia controlled with
atorvastatin. His laboratory studies are notable for an elevated ferritin, and after demonstration of a
mutation of the HFE gene, he is diagnosed with hemochromatosis. Which of the following statements
regarding his joint abnormalities is true?
A. The second and third finger MCPs are also typically involved in osteoarthritis.
B. Arthropathy is unlikely related to hemochromatosis.
C. Arthropathy may progress with phlebotomy.
D. Arthropathy occurs in less than 20% of patients with hemochromatosis.
E. Radiographs are likely to show erosions in the MCPs.

IX-80. A 64-year-old woman is seen by her primary care physician complaining of hip pain for about 1
week. She localizes the pain to the lateral aspect of her right hip and describes it as sharp. It is worse
with movement, and she finds it difficult to lie on her right side. The pain began soon after the
patient was planting her garden. She has a medical history of obesity, osteoarthritis of the knees, and
hypertension. Her medications include losartan 50 mg daily and hydrochlorothiazide 25 mg daily.
For the pain, she has taken ibuprofen 600 mg as needed with mild to moderate relief of pain. On
physical examination, the patient is not febrile and her vital signs are unremarkable. On examination
of the hip, pain is elicited with external rotation and resisted abduction of the hip. Direct palpation
over the lateral aspect of the upper portion of the femur near the hip joint reproduces the pain. What
is the most likely diagnosis in this patient?
A. Avascular necrosis of the hip
B. Iliotibial band syndrome
C. Meralgia paresthetica
D. Septic arthritis
E. Trochanteric bursitis

IX-81. A 32-year-old woman is seen in clinic with a complaint of left knee pain. She enjoys running long
distances and is currently training for a marathon. She is running on average 30–40 miles weekly.
She currently is experiencing an aching pain on the lateral aspect of her left knee. There is a burning
sensation that also continues up the lateral aspect of her thigh. She denies any injury to her knee, and
she has not felt that it was hot or swollen. She is otherwise healthy and takes no medications other
than herbal supplements. Physical examination of the knee reveals point tenderness over the lateral
femoral condyle that is worse with flexing the knee. The patient is asked to lie on her right side with
her right knee and hip flexed at 90 degrees. Her left leg is extended at the hip and slowly lowered
into adduction behind the bottom leg, reproducing the patient’s left knee pain. All of the following
treatments can be recommended for this patient EXCEPT:
A. Assessment of the patient’s running shoes to ensure a proper fit
B. Glucocorticoid injection so as not to interfere with the patient’s continued preparation for the
upcoming marathon
C. Ibuprofen 600–800 mg every 6 hours as needed for pain
D. Referral for physical therapy
E. Referral for surgical release if conservative therapy fails

IX-82. A 58-year-old woman presents complaining of right shoulder pain. She does not recall any prior
injury but notes that she feels that the shoulder has been getting progressively more stiff over the last
several months. She previously had several episodes of bursitis of the right shoulder that were treated
successfully with nonsteroidal anti-inflammatory drugs and steroid injections. The patient’s past
medical history is also significant for diabetes mellitus, for which she takes metformin and glyburide.
On physical examination, the right shoulder is not warm or red but is tender to touch. Passive and
active range of motion are limited in flexion, extension, and abduction. A right shoulder radiogram
shows osteopenia without evidence of joint erosion or osteophytes. What is the most likely diagnosis?
A. Adhesive capsulitis
B. Avascular necrosis
C. Bicipital tendinitis
D. Osteoarthritis
E. Rotator cuff tear

IX-83. A 32-year-old woman presents to clinic with right thumb and wrist pain that has worsened over
several weeks. She has pain when she pinches her thumb against her other fingers. Her only other
history is that she is a new mother with an 8-week-old infant at home. On physical examination, she
has mild swelling and tenderness over the radial styloid process, and pain is elicited when she places
her thumb in her palm and grasps it with her fingers. A Phalen maneuver is negative. Which
condition is most likely?
A. Carpal tunnel syndrome
B. De Quervain tenosynovitis
C. Gouty arthritis of the first metacarpophalangeal joint
D. Palmar fasciitis
E. Rheumatoid arthritis

IX-84. You are evaluating Ms. Rumpulo, a 42-year-old woman who complains of pain on the underside
of her right heel that is excruciating in the morning when she first walks from bed to the bathroom.
The pain improves somewhat during the morning but again worsens mid-day particularly when
climbing stairs. She has a past medical history of hypertension, smokes one pack per day of
cigarettes, and works as a waitress at a diner. Medications include hydrochlorothiazide and oral
contraceptives. Physical examination is unremarkable except for flat feet and focal tenderness on the
bottom of the right heel. There is no tenderness at the ankle or calf, and the diameters of the lower
legs are equivalent. A radiograph of the right heel and ankle shows only heel spurs. All of the
following statements regarding Ms. Rumpulo’s condition are true EXCEPT:
A. Heel spurs are not diagnostic.
B. Local glucocorticoid injection incurs a risk of plantar facial rupture.
C. Oral contraceptives and smoking are risk factors.
D. Orthotic shoe implants may be beneficial.
E. The prognosis for improvement is good.

ANSWERS

IX-1. The answer is A. (Chap. 372e) The innate immune system is phylogenetically the oldest form of
immunologic defense system, inherited from invertebrates. This defense system uses germline-
encoded proteins to recognize pathogen-associated molecular patterns. Cells of the innate
immune system include macrophages, dendritic cells, and natural killer lymphocytes. The critical
components of the innate immune system include recognition by germline-encoded host
molecules and recognition of key microbe virulence factors, but no recognition of self-molecules
and of benign foreign molecules or microbes. Adaptive immunity is found only in vertebrate
animals and is based on generation of antigen receptors on T and B lymphocytes by gene
rearrangements, such that individual T or B cells express unique antigen receptors on their
surface capable of recognizing diverse environmental antigens.

IX-2. The answer is A. (Chap. 372e) Complement activity, which results from the sequential
interaction of a large number of plasma and cell membrane proteins, plays an important role in
the inflammatory response. The classic pathway of complement activation is initiated by an
antibody-antigen interaction. The first complement component (C1, a complex composed of
three proteins) binds to immune complexes with activation mediated by C1q. Active C1 then
initiates the cleavage and concomitant activation of components C4 and C2. The activated C1 is
destroyed by a plasma protease inhibitor termed C1 esterase inhibitor. This molecule also
regulates clotting factor XI and kallikrein. Patients with a deficiency of C1 esterase inhibitor may
develop angioedema, sometimes leading to death by asphyxia. Attacks may be precipitated by
stress or trauma. In addition to low antigenic or functional levels of C1 esterase inhibitor,
patients with this autosomal dominant condition may have normal levels of C1 and C3 but low
levels of C4 and C2. Danazol therapy produces a striking increase in the level of this important
inhibitor and alleviates the symptoms in many patients. An acquired form of angioedema caused
by a deficiency of C1 esterase inhibitor has been described in patients with autoimmune or
malignant disease.

IX-3. The answer is C. (Chap. 373e) The human major histocompatibility complex (MHC) genes are
located on a 4-megabase region on chromosome 6. The major function of the MHC complex
genes is to produce proteins that are important in developing immunologic specificity through
their role in binding antigen for presentation to T cells. This process is nonspecific, and the
ability of a human leukocyte antigen (HLA) molecule to bind to a particular protein depends on
the molecular fit between the amino acid sequence of a particular protein and the corresponding
domain on the MHC molecule. Once a peptide has bound, the MHC-peptide complex binds to the
T-cell receptor, after which the T cell must determine if an immune response should be
generated. If an antigen is similar to an endogenous protein, the potential antigen will be
recognized as a self-peptide, and tolerance to the antigen will be continued. The MHC I and II
complexes have been implicated in the development of many autoimmune diseases, which occur
when T cells fail to recognize a peptide as a self-peptide and an immune response is allowed to
develop. MHC I and II genes also play a major role in tissue compatibility for transplantation and
are important in generating immune-mediated rejection. The other options listed as answers
refer to functions of immunoglobulins. The variable region of the immunoglobulin is a B-cell–
specific response to an antigen to promote neutralization of the antigen through agglutination
and precipitation. The constant region of the immunoglobulin is able to nonspecifically activate
the immune system through complement activation and promotion of phagocytosis by
neutrophils and macrophages.

IX-4. The answer is E. (Chap. 374) Hundreds of gene products have been characterized as effectors
or mediators of the immune system (Chap. 372e). Whenever the expression or function of one of
these products is genetically impaired (provided the function is nonredundant), a primary
immunodeficiency (PID) occurs. PIDs are genetic diseases with primarily Mendelian inheritance.
More than 250 conditions have now been described, and deleterious mutations in approximately
210 genes have been identified. The overall prevalence of PIDs has been estimated in various
countries at 5 per 100,000 individuals; however, given the difficulty in diagnosing these rare and
complex diseases, this figure is probably an underestimate. PIDs can involve all possible aspects
of immune responses, from innate through adaptive, cell differentiation, and effector function
and regulation. For the sake of clarity, PIDs should be classified (see Table IX-4) according to (1)
the arm of the immune system that is defective and (2) the mechanism of the defect (when
known). The consequences of PIDs vary widely as a function of the molecules that are defective.
This concept translates into multiple levels of vulnerability to infection by pathogenic and
opportunistic microorganisms, ranging from extremely broad (as in severe combined
immunodeficiency [SCID]) to narrowly restricted to a single microorganism (as in Mendelian
susceptibility to mycobacterial disease [MSMD]). The locations of the sites of infection and the
causal microorganisms involved will thus help physicians arrive at proper diagnoses. PIDs can
also lead to immunopathologic responses such as allergy (as in Wiskott-Aldrich syndrome),
lymphoproliferation, and autoimmunity. A combination of recurrent infections, inflammation,
and autoimmunity can be observed in a number of PIDs, thus creating obvious therapeutic
challenges. The most frequent symptom prompting the diagnosis of a PID is the presence of
recurrent or unusually severe infections. Infections of the respiratory tract (bronchi, sinuses)
mostly suggest a defective antibody response. In general, invasive bacterial infections can result
from complement deficiencies, signaling defects of innate immune responses, asplenia, or
defective antibody responses. Viral infections, recurrent Candida infections, and opportunistic
infections are generally suggestive of impaired T-cell immunity. Skin infections and deep-seated
abscesses primarily reflect innate immune defects (such as chronic granulomatous disease);
however, they may also appear in the autosomal dominant hyper-immunoglobulin E (IgE)
syndrome. Finally, some PIDs increase the risk of cancer, notably but not exclusively
lymphocytic cancers (e.g., lymphoma).

TALBLE IX-4 Classification of Primary Immune Deficiency Diseases


IX-5. The answer is B. (Chap. 374) This patient has axillary folliculitis, an infection of the hair
follicles. Based on his history, including recurrent infections with the catalase-positive organism
Serratia marcescens, he most likely has chronic granulomatous disease (CGD). CGDs are
characterized by impaired phagocytic killing of microorganisms by neutrophils and
macrophages. About 70% of cases are associated with X-linked recessive inheritance versus
autosomal inheritance in the remaining 30%. CGD causes deep tissue bacterial and fungal
abscesses in macrophage-rich organs such as the skin, lymph nodes, liver, and lungs. Recurrent
skin infections, such as folliculitis, are common and can prompt an early diagnosis of CGD. The
infectious agents are typically catalase-positive bacteria (such as Staphylococcus aureus and
Serratia marcescens) but also include Burkholderia cepacia, pathogenic mycobacteria (in certain
regions of the world), and fungi (mainly filamentous molds, such as Aspergillus). CGD is caused
by defective production of reactive oxygen species (ROS) in the phagolysosome membrane
following phagocytosis of microorganisms. Diagnosis of CGD is based on assays of ROS
production in neutrophils and monocytes such as the dihydrorhodamine (DHR) fluorescence or
nitroblue tetrazolium (NBT) assays. CGD is a granulomatous disease with macrophage-rich
granulomas in the liver, spleen, and other organs. These are sterile granulomas that cause
disease by obstruction (bladder, pylorus, etc.) or inflammation (colitis, restrictive lung disease).
The treatment of bacterial infections is generally based on combination therapy with antibiotics
that are able to penetrate into cells. The treatment of fungal infections requires aggressive, long-
term use of antifungals. Inflammatory/granulomatous lesions are usually steroid sensitive;
however, glucocorticoids often contribute to the spread of infections. The treatment of CGD
mostly relies on preventing infections. It has been unambiguously demonstrated that
prophylactic usage of trimethoprim/sulfamethoxazole is both well tolerated and highly effective
in reducing the risk of bacterial infection. Daily administration of azole derivatives (notably
itraconazole) also reduces the frequency of fungal complications. Human stem cell
transplantation is an established curative approach for CGD; however, the risk-benefit ratio must
be carefully assessed on a case-by-case basis. Gene therapy approaches are also being evaluated.

IX-6. The answer is B. (Chap. 376) The patient has classic symptoms of angioedema with rapid onset
of facial swelling often involving the lips, frequently with preceding limb symptoms.
Angioedema and urticaria are grouped by the underlying etiology. In this case, angiotensin-
converting enzyme (ACE) inhibitor use is associated with increased levels of bradykinin, which
in a predisposed individual can result in angioedema. Hereditary angioedema is associated with
chronically depressed levels of C1 inhibitor, which is involved in the degradation of bradykinin.
IgE-mediated angioedema occurs due to specific antigen sensitivity, and complement-mediated
disease may be due to vasculitis, serum sickness, or reactions to blood products. Finally,
nonimmunologic causes of angioedema include direct mast cell–releasing agents such as opiates
and agents that alter arachidonic acid metabolism, most commonly nonsteroidal anti-
inflammatory drugs (NSAIDs). IgE levels are not elevated in bradykinin-mediated angioedema.
Because of the potentially life-threatening nature of disease, rechallenge with a second ACE
inhibitor is not 1recommended.

IX-7. The answer is D. (Chap. 376) This patient presents with symptoms of cold urticaria, an IgE-
dependent urticarial reaction to cold exposure. After exposure to cold, urticarial lesions appear
in exposed areas and usually last for <2 hours. Histologic examination of the urticarial lesion
would demonstrate mast cell degranulation with edema of the dermis and subcutaneous tissues.
In experimental exposure to a cold challenge such as an ice water bath, elevated levels of
histamine in venous blood may be demonstrated if assessed in the extremity exposed to a cold
environment, whereas the histamine levels would be normal in a nonexposed extremity. The
appearance of a linear wheal after a firm stroke is indicative of dermatographism. This condition
can be seen in 1%–4% of the population and is often found in individuals with cold urticaria. In
general, cold urticaria is a localized process without adverse consequences. However, vascular
collapse may occur if an individual is submerged in cold water. Many individuals request
treatment because they are embarrassed by their condition or are symptomatic from the
recurrent urticaria and pruritus. Treatment with H1 histamine receptor blockers is usually
adequate for symptom control. Cyproheptadine or hydroxyzine can be added to therapy if H1
antihistamines are inadequate. In this patient, there is a clear precipitant for developing urticaria
—cold exposure. Thus, no other evaluation is necessary. In the evaluation and management of
chronic urticaria, identification and elimination of precipitating factors are important. Possible
etiologic factors include foods, pollens, molds, and medications. In this case, the urticaria
predates the use of oral contraceptive medications; thus, stopping oral contraceptives would be
unlikely to be helpful. Assessment of antithyroglobulin and antimicrosomal antibodies can be
helpful in individuals with chronic urticaria in whom a cause is not otherwise identified.
Deficiency of C1 or the presence of a C1 inhibitor presents as recurrent angioedema rather than
urticaria.

IX-8. The answer is E. (Chap. 376) Allergic rhinitis is a common problem in the United States and
North America. It is estimated that approximately one in five individuals experiences allergic
rhinitis. The incidence is greatest in childhood and adolescence, and the symptoms tend to
regress with aging. Complete remissions, however, are uncommon. Many individuals experience
seasonal symptoms only. These symptoms are due to pollen production by weeds, grasses, and
trees that are dependent on wind currents, rather than insects, for cross-pollination. The timing
of the pollination events predicts seasonal severity of symptoms and varies little from year to
year within a particular locale. Based on this pattern, one is able to predict which allergens are
most likely responsible for a patient’s symptoms. In the temperate regions of North America,
trees pollinate in the spring, and ragweed pollinates in the fall. Grasses are responsible for
seasonal allergic symptoms in the summer months. Mold allergens can have a variable pattern of
symptoms, depending on climactic conditions that allow them to sporulate. Perennial rhinitis
does not have a seasonal pattern and is more continually present. Allergens that cause perennial
rhinitis include animal dander, dust, and cockroach-derived proteins.

IX-9. The answer is D. (Chap. 376) There is no convincing evidence that age, sex, race, or geographic
location predisposes a human to anaphylaxis except through exposure to specific immunogens.
According to most studies, atopy does not predispose individuals to anaphylaxis from penicillin
therapy or venom of a stinging insect but is a risk factor for allergens in food or latex. Risk
factors for a poor outcome, however, include older age, use of β-blockers, and the presence of
preexisting asthma. Individuals differ in the time of appearance of symptoms and signs, but the
hallmark of the anaphylactic reaction is the onset of some manifestation within seconds to
minutes after introduction of the antigen. Early recognition of an anaphylactic reaction is
mandatory, because death can occur within minutes to hours after the first symptoms.

Mild symptoms such as pruritus and urticaria can be controlled by administration of 0.3–0.5
mL of 1:1000 (1 mg/mL) epinephrine subcutaneously (SC) or intramuscularly (IM), with repeated
doses as required at 5- to 20-minute intervals for a severe reaction. The failure to use epinephrine
within the first 20 minutes of symptoms is a risk factor for poor outcome in studies of
anaphylaxis to food. An intravenous (IV) infusion should be initiated to provide a route for
administration of 2.5 mL epinephrine, diluted 1:10,000, at 5- to 10-minutes intervals, volume
expanders such as normal saline, and vasopressor agents such as dopamine if intractable
hypotension occurs. Replacement of intravascular volume due to postcapillary venular leakage
may require several liters of saline. Epinephrine provides both α- and β-adrenergic effects,
resulting in vasoconstriction, bronchial smooth muscle relaxation, and attenuation of enhanced
venular permeability. Ancillary agents such as the antihistamine diphenhydramine, 50–100 mg
IM or IV, and aminophylline, 0.25–0.5 g IV, are appropriate for urticaria-angioedema and
bronchospasm, respectively. IV glucocorticoids (0.5–1 mg/kg of methylprednisolone) are not
effective for the acute event but may alleviate later recurrence of bronchospasm, hypotension, or
urticaria.

IX-10. The answer is E. (Chap. 377e) Derangements of normal processes may predispose to the
development of autoimmunity. In general, these abnormal responses require both an exogenous
trigger, such as infection (bacterial or viral) or cigarette smoking, and the presence of
endogenous abnormalities in the cells of the immune system. One of the best examples of
autoreactivity and autoimmune disease resulting from molecular mimicry is rheumatic fever, in
which antibodies to the M protein of streptococci cross-react with myosin, laminin, and other
matrix proteins as well as with neuronal antigens. Deposition of these autoantibodies in the heart
initiates an inflammatory response, whereas their penetration into the brain can result in
Sydenham chorea. Molecular mimicry between microbial proteins and host tissues has been
reported in type 1 diabetes mellitus, rheumatoid arthritis, celiac disease, and multiple sclerosis.

TALBLE IX-10 Mechanisms of Autoimmunity


IX-11. The answer is D. (Chap. 377e) The mechanisms of tissue injury in autoimmune diseases can be
divided into antibody-mediated and cell-mediated processes. Autoantibodies can interfere with
normal physiologic functions of cells or soluble factors. Autoantibodies to hormone receptors can
lead to stimulation of cells or to inhibition of cell function through interference with receptor
signaling. For example, long-acting thyroid stimulators—autoantibodies that bind to the receptor
for thyroid-stimulating hormone (TSH)—are present in Graves disease and function as agonists,
causing the thyroid to respond as if there were an excess of TSH. In contrast, Hashimoto
thyroiditis is due to antibody-dependent cellular cytotoxicity due to antibodies directed against
thyroid peroxidase.

IX-12. The answer is C. (Chap. 378) Antinuclear antibodies are nearly ubiquitous in patients with
systemic lupus erythematosus (SLE), with demonstration in 90% of affected patients. There are
many other antibodies that can be demonstrated. The next most common antibodies are anti-
dsDNA and anti-histone. Anti-dsDNA is very specific to SLE and may correlate with disease
activity, nephritis, and vasculitis. Antiphospholipid antibodies can be demonstrated in about half
of affected patients, whereas the remainder is present in less than half of SLE cases.
Antierythrocyte antibodies are present in approximately 60% of SLE cases and can be measured
by a direct Coombs test. In contrast, anti-La antibodies directed against the 47-kDa protein
complexes to hY RNA is rare, present only in 10% of cases of SLE. It is associated with a
decreased risk for nephritis.

IX-13. The answer is B. (Chap. 378) There are well-published, strict diagnostic criteria for SLE. They
include four or more of the following criteria from the table below.
The patient described does not meet the arthritis criteria; her only criteria are oral ulcers and
weakly positive ANA.
IX-14. The answer is A. (Chap. 378) The patient has Libman-Sacks endocarditis associated with her
SLE. This results in fibrinous endocarditis and can lead to valvular insufficiencies, most often
mitral or aortic, or embolism. It is not generally found with concomitant pericarditis, although
this is another common cardiac manifestation of systemic lupus erythematosus. Although
glucocorticoids and anti-inflammatory therapies have no proven benefit in this condition, they
are often used in conjunction with supportive care. Because Libman-Sacks endocarditis is a
culture-negative endocarditis and is not thought to be due to microbial infection, blood cultures
will not be positive.
IX-15. The answer is D. (Chap. 378) SLE is a multisystem disease with diverse organ involvement and
multiple different manifestations within an organ system. The most common system to be
involved is the musculoskeletal system, with 95% of patients having involvement, most
commonly as arthralgias or myalgias. Arthritis is also common and is one of the diagnostic
criteria for SLE. Cutaneous and hematologic disease occurs in approximately 80%–85% of
patients. Neurologic and cardiopulmonary disease affects approximately 60% of patients,
whereas renal and gastrointestinal disease occurs in <50% of cases.

IX-16. The answer is C. (Chap. 378) This patient is presenting with acute lupus nephritis with
evidence of hematuria, proteinuria, and an acute rise in creatinine. Together with infection,
nephritis is the most common cause of mortality in the first decade after diagnosis of SLE and
warrants prompt immunosuppressive therapy. It is important to assess for other potentially
reversible causes of acute renal insufficiency, but this patient is not otherwise acutely ill and is
taking no medications that would cause renal failure. The urinalysis shows evidence of active
nephritis with hematuria and proteinuria. Even in the absence of red blood cell (RBC) casts,
therapy should not be withheld to await biopsy results in someone with a known diagnosis of
SLE with consistent clinical presentation and urinary findings. This patient also has other risk
factors known to predict the development of lupus nephritis, including high titers of anti-dsDNA
and African American race. The mainstay of treatment for any life-threatening or organ-
threatening manifestation of SLE is high-dose systemic glucocorticoids. Addition of cytotoxic or
other immunosuppressive agents (cyclophosphamide, azathioprine, mycophenolate mofetil) is
recommended to treat serious complications of SLE, but their effects are delayed for 3–6 weeks
after initiation of therapy, whereas the effects of glucocorticoids begin within 24 hours. Thus,
these agents alone should not be used to treat acute serious manifestations of SLE. The choice of
cytotoxic agent is at the discretion of the treating physician. Cyclophosphamide in combination
with steroid therapy has been demonstrated to prevent development of end-stage renal disease
better than steroids alone. Likewise, mycophenolate also prevents development of end-stage
renal disease in combination with glucocorticoids, and some studies suggest that African
Americans have a greater response to mycophenolate than to cyclophosphamide. Plasmapheresis
is not indicated in the treatment of lupus nephritis but is useful in cases of severe hemolytic
anemia or thrombotic thrombocytopenic purpura associated with SLE. Finally, this patient has
no acute indication for hemodialysis and, with treatment, may recover renal function.

IX-17. The answer is A. (Chap. 379) The patient has multiple clinical manifestations of arterial
thrombosis in her hand and brain and likely had placental insufficiency in the three prior
pregnancies, which makes the possibility of antiphospholipid antibody syndrome likely. In
addition, she has evidence of acute kidney injury, suggesting multisystem disease.
Thrombocytopenia may be due to hemolytic anemia, but the absence of schistocytes makes it
less likely that she has thrombotic thrombocytopenic purpura. Although magnetic resonance
imaging (MRI) of her brain and extremity duplex may confirm the presence of thrombosis, these
will not diagnose antiphospholipid antibody syndrome. An anticardiolipin antibody screening
panel will look for evidence of antibodies directed against cardiolipin and β-2 glycoprotein I.
Additional testing for lupus anticoagulant determined by clotting assays such as the Russel viper
venom time, false-positive rapid plasma reagin (RPR), and the activated partial thromboplastin
time (aPTT) may also be useful. ANA is likely to be positive given the common overlap with SLE,
but is nonspecific.

IX-18. The answer is D. (Chap. 379) This patient has a typical presentation of antiphospholipid
syndrome (APS) with a deep venous thrombosis, history of spontaneous abortion, and isolated
elevated aPTT due to a lupus anticoagulant. Additional clinical features of APS involving the
arterial or venous circulation include livedo reticularis (24%), pulmonary embolism (14%),
stroke (20%), transient ischemic attack (10%), myocardial infarction (10%), migraine (20%),
preeclampsia (10%), thrombocytopenia (30%), and autoimmune hemolytic anemia (10%).
Laboratory criteria include demonstration of lupus anticoagulant (elevated aPTT that does not
correct on mixing), in conjunction with the presence of anticardiolipin and/or anti-β-2
glycoprotein I on two occasions 3 month apart. After diagnosis of a thrombotic event due to APS,
patients should receive warfarin for life with a goal international normalized ratio (INR) of 2.5–
3.5 alone or in combination with daily aspirin. During pregnancy, patients should receive
heparin plus aspirin. Patients who develop recurrent thrombosis while on effective
anticoagulation may benefit from a 5-day infusion of IV γ-globulin or 4 weeks of rituximab
therapy. The optimal therapy for patients with APS without a thrombotic event is not known;
however, daily aspirin (80 mg) protects patients with SLE and antiphospholipid antibodies from
thrombotic events. Warfarin for 3 months with INR goal of 2.0–3.0 is recommended therapy for
deep vein thrombosis (DVT) with a known reversible precipitating event. Warfarin for 6–12
months with an INR goal of 2.0–3.0 is recommended therapy for first episode of idiopathic DVT.

IX-19. The answer is E. (Chap. 379) Patients with APS often possess antibodies recognizing Treponema
pallidum PL/cholesterol complexes, which are detected as biologic false-positive serologic tests
for syphilis (BFP-STS) and Venereal Disease Research Laboratory (VDRL) tests. If syphilis is
suspected, a specific direct treponemal test, such as fluorescent treponemal antibody absorption
(FTA-ABS) should be obtained.

IX-20. The answer is E. (Chap. 380) Once the disease process of rheumatoid arthritis is established,
the most common joints of involvement are the wrists, metacarpophalangeal joints, and proximal
interphalangeal joints. Distal interphalangeal joint involvement is rarely due to rheumatoid
arthritis and more often due to coexisting osteoarthritis.

IX-21. The answer is C. (Chap. 380) There is potential involvement of multiple organ systems in
rheumatoid arthritis (RA). The most common pulmonary complication is pleural effusion that is
typically exudative and presents with chest pain and dyspnea. RA is associated with a form of
diffuse interstitial lung disease that may present with dyspnea and bilateral interstitial infiltrates
that may be so extensive as to develop into a honeycomb pattern. Pulmonary nodules associated
with rheumatoid arthritis may be solitary or multiple. They often occur in conjunction with
cutaneous nodules. Bronchiectasis and respiratory bronchiolitis may also be due to rheumatoid
arthritis. Many of these manifestations respond to immunosuppressive therapy. Lobar infiltrate
has not been described to be caused by RA and is more commonly caused by an acute infectious
etiology, often as a complication of RA immunosuppressive therapy.

IX-22. The answer is A. (Chap. 380) Joint imaging is a critical tool for both diagnosis and monitoring
of disease status in RA. Plain radiographs, because of their ready availability and ease of film
comparison, are most commonly ordered. The earliest clinical sign of RA is juxta-articular
osteopenia, although this may be difficult to appreciate on newer, digitized films. Other findings
include soft tissue swelling, symmetric joint space loss, and subchondral erosions most frequently
in the wrists, metacarpophalangeal and proximal interphalangeal joints, and
metatarsophalangeal joint.

IX-23. The answer is E. (Chap. 380) Anemia is common in RA and parallels the degree of
inflammation as measured by C-reactive protein or erythrocyte sedimentation rate (ESR). Felty
syndrome, typically occurring in late-stage poorly controlled disease, is characterized by the
triad of neutropenia, splenomegaly, and rheumatoid nodules. Rheumatoid vasculitis is not
common and typically occurs in long-standing disease. It is associated with
hypocomplementemia. The cutaneous signs are typical of vasculitic lesions with palpable
purpura, digital infarcts, livedo reticularis, and ulcers. Clinical manifestations of pericarditis
occur in 10% of patients with echocardiographic or autopsy findings in about half of those cases.
Secondary Sjögren syndrome manifest as keratoconjunctivitis sicca or xerostomia occurs in
approximately 10% of patients with RA. RA also appears to increase the risk of developing B-cell
lymphoma by two- to fourfold compared with the general population. The risk of lymphoma
appears to correlate with high levels of disease activity or the presence of Felty syndrome.
Platelet counts in RA are typically elevated in association with the acute-phase response of
inflammation. Immune thrombocytopenia is rare.

IX-24. The answer is D. (Chap. 380) The therapy of RA has changed dramatically in the past two
decades with the development of drugs that modify the disease course of RA. Methotrexate is the
disease-modifying antirheumatic drug (DMARD) of first choice for treatment of early RA. Other
conventional DMARDs include hydroxychloroquine, sulfasalazine, and leflunomide. Leflunomide,
an inhibitor of pyrimidine synthesis, is efficacious as a single agent or in combination with
methotrexate. Hydroxychloroquine and sulfasalazine are typically reserved for mild disease. The
biologic DMARDs have dramatically improved the treatment of RA in the past decade. There are
currently five anti–tumor necrosis factor (TNF) agents, including infliximab, approved for use in
patients with RA. Rituximab, an anti-CD20 antibody, is approved for refractory RA in
combination with methotrexate. It is more efficacious in seropositive than seronegative patients.
Other biologics approved for use in RA include anakinra (interleukin [IL]-1 receptor antagonist),
abatacept (CD28/CD80/86 antagonist), and tocilizumab (IL-6 antagonist). NSAIDs, including
naproxen, were formerly used as core RA therapy. However, they are now used as adjunctive
treatment for symptom management. They are not considered DMARDs.

IX-25. The answer is E. (Chap. 381) Felty syndrome is defined by the clinical triad of neutropenia,
splenomegaly, and nodular RA and is seen in less than 1% of patients, although its incidence
appears to be declining in the face of more aggressive treatment of the joint disease. It typically
occurs in the late stages of severe RA and is more common in whites than other racial groups. T-
cell large granular lymphocytic leukemia (T-LGL) may have a similar clinical presentation and
often occurs in association with RA. T-LGL is characterized by a chronic, indolent clonal growth
of LGL cells, leading to neutropenia and splenomegaly. As opposed to Felty syndrome, T-LGL
may develop early in the course of RA. Leukopenia apart from these disorders is uncommon and
most often due to drug therapy.

IX-26. The answer is C. (Chap. 380) Large cohort studies have shown a two- to fourfold increased risk
of lymphoma in RA patients compared with the general population. The most common
histopathologic type of lymphoma is a diffuse large B-cell lymphoma. The risk of developing
lymphoma increases if the patient has high levels of disease activity or Felty syndrome.

IX-27. The answer is D. (Chap. 381) Acute rheumatic fever is almost universally due to group A
streptococcal disease in present time, though virtually all streptococcal disease may be capable
of precipitating rheumatic fever. Although skin infections may be associated with rheumatic
fever, far and away the most common presentation is with preceding pharyngitis. There is a
latent period of approximately 3 weeks from an episode of sore throat to presentation of acute
rheumatic fever. The most common manifestations are fever and polyarthritis, with polyarthritis
present in 60%–75% of cases. Carditis may also be present, although somewhat less frequently
(50%–60% of cases). Chorea and indolent carditis may have a subacute presentation. Chorea is
present in 2%–30% of affected individuals, whereas erythema marginatum and subcutaneous
nodules are rare. Sixty percent of patients with acute rheumatic fever progress to rheumatic
heart disease with the endocardium, pericardium, and myocardium all potentially involved. All
patients with acute rheumatic fever should receive antibiotics sufficient to treat the precipitating
group A streptococcal infection.

IX-28. The answer is D. (Chap. 381) This patient has a history suggestive of recurrent bouts of acute
rheumatic fever (ARF) with evidence of mitral regurgitation, mitral stenosis, and aortic
regurgitation on physical examination. This and the presence of atrial fibrillation imply severe
rheumatic heart disease. Risk factors for this condition include poverty and crowded living
conditions. As a result, ARF is considerably more common in the developing world. Daily aspirin
is the treatment of choice for the migratory large-joint arthritis and fever that are common
manifestations of ARF. Practitioners sometimes use steroids during acute bouts of carditis to
quell inflammation, although this remains a controversial practice and has no role between flares
of ARF. Secondary prophylaxis with either daily oral penicillin or, preferably, monthly IM
injections is considered the best method to prevent further episodes of ARF and, therefore,
prevent further valvular damage. Primary prophylaxis with penicillin on an as-needed basis is
equally effective for preventing further bouts of carditis. However, most episodes of sore throat
are too minor for patients to present to a physician. Therefore, secondary prophylaxis is
considered preferable in patients who already have severe valvular disease. Doxycycline is not a
first-line agent for group A Streptococcus, the pathogen that incites ARF.

IX-29. The answer is A. (Chap. 381) There is a latent period of approximately 3 weeks (1–5 weeks)
between the precipitating group A streptococcal infection and the appearance of the clinical
features of ARF. The exceptions are chorea and indolent carditis, which may follow prolonged
latent periods lasting up to 6 months. Although many patients report a prior sore throat, the
preceding group A streptococcal infection is commonly subclinical; in these cases, it can only be
confirmed using streptococcal antibody testing. The most common clinical features are
polyarthritis (present in 60%–75% of cases) and carditis (50%–60%). The prevalence of chorea
in ARF varies substantially between populations, ranging from <2% to 30%. Erythema
marginatum and subcutaneous nodules are now rare, being found in <5% of cases.

IX-30. The answer is A. (Chap. 382) The prognosis for patients with scleroderma renal disease is poor.
In scleroderma renal crisis patients, prompt treatment with an ACE inhibitor may reverse acute
renal failure. In recent studies, the initiation of ACE inhibitor therapy resulted in 61% of patients
having some degree of renal recovery and not needing chronic dialysis support. The survival rate
is estimated to be 80%–85% at 8 years. Among patients who needed dialysis, when treated with
ACE inhibitors, over 50% were able to discontinue dialysis after 3–18 months. Therefore, ACE
inhibitors should be used even if the patient requires dialysis support.

IX-31. The answer is C. (Chap. 382) Virtually every organ can be clinically affected with cutaneous
systemic sclerosis (SSc). Most patients with SSc can be classified as either limited (lcSSc) or
diffuse (dcSSc). Although stratification of SSc patients into diffuse and limited cutaneous subsets
is useful, disease expression is far more complex, and several distinct endophenotypes exist
within each subset. In general, pulmonary parenchymal involvement is more common in patients
with dcSSc than lcSSc. See Figure IX-31.

FIGURE IX-31

IX-32. The answer is E. (Chap. 382) Patients who have lcSSc coexisting with features of SLE,
polymyositis, and rheumatoid arthritis may have mixed connective tissue disease (MCTD). This
overlap syndrome is generally associated with the presence of high titers of autoantibodies to
U1-RNP. Laboratory evaluation indicates features of inflammation with elevated ESR and
hypergammaglobulinemia. Although anti–U1-RNP antibodies are detected in the serum in high
titers, SSc-specific autoantibodies are not found. In contrast to SSc, patients with MCTD often
show a good response to treatment with glucocorticoids, and the long-term prognosis is better
than that of SSc. Anticentromere antibodies are typically associated with lcSSc.

IX-33. The answer is D. (Chap. 383) The patient presented with classic symptoms for Sjögren
syndrome including dry mouth and eyes. This condition may be primary, as in this case, or
secondary in association with another connective tissue disease such as scleroderma or
rheumatoid arthritis. Many autoantibodies may be demonstrated in the serum of patients with
Sjögren syndrome, including antibodies to Ro/SS-A or La/SS-B. Sialometry will demonstrate
decreased production of saliva and MRI or magnetic resonance sialography of the major salivary
glands may also demonstrate abnormalities. Ocular involvement with decreased tear production
is demonstrated by the Schirmer I test. Scl-70 antibody is associated with scleroderma and
should not be positive in primary Sjögren syndrome.

IX-34. The answer is D. (Chap. 383) Lymphoma is well known to develop specifically in the late stage
of Sjögren syndrome. Common manifestations of this malignant condition include persistent
parotid gland enlargement, purpura, leukopenia, cryoglobulinemia, and low C4 complement
levels. Most of the lymphomas are extranodal, marginal zone B cell, and low grade. Low-grade
lymphomas may be detected incidentally during a labial biopsy. Mortality is higher in patients
with concurrent B symptoms (fevers, night sweats, and weight loss), a lymph node mass >7 cm,
and a high or intermediate histologic grade.

IX-35. The answer is A. (Chap. 383) Although Sjogren syndrome most commonly affects the eyes and
mouth, there are a number of common extraglandular sites of involvement. The most common is
arthritis or arthralgias that complicate up to 60% of cases. Raynaud phenomenon is the second
most common extraglandular site. Lung involvement and vasculitis are found in less than 20% of
patients. Lymphoma, although a concerning and highly morbid complication, is relatively rare,
affecting only 6% of Sjögren patients.

IX-36. The answer is B. (Chap. 383) Primary Sjögren syndrome is diagnosed if (1) the patient presents
with eye and/or mouth dryness, (2) eye tests disclose keratoconjunctivitis sicca, (3) mouth
evaluation reveals the classic manifestations of the syndrome, and/or (4) the patient’s serum
reacts with Ro/SS-A and/or La/SS-B autoantigens. Labial biopsy is needed when the diagnosis is
uncertain or to rule out other conditions that may cause dry mouth or eyes or parotid gland
enlargement. See Table IX-36 for the differential diagnosis of Sjögren syndrome.

TALBLE IX-36 Differential Diagnosis of Sjögren Syndrome


IX-37. The answer is D. (Chap. 384) Ankylosing spondylitis is closely correlated with the presence of
the histocompatibility antigen HLA-B27. In North American whites, the prevalence of B27 is 7%,
but in patients with ankylosing spondylitis, it is 90%. Not all persons with B27 develop
ankylosing spondylitis; the disease is only present in 1%–6% of B27-positive individuals.

IX-38. The answer is A. (Chap. 384) Although the most serious spine complication of ankylosing
spondylitis is fracture, there are a number of important extra-articular manifestations. Anterior
uveitis is the most common, occurring in 40% of patients with ankylosing spondylitis.
Inflammatory bowel disease has been reported to be frequently present. Less common
complications include aortic insufficiency, third-degree heart block, pulmonary nodules and
upper lobe fibrosis, cardiac dysfunction, retroperitoneal fibrosis, prostatitis, and amyloidosis.

IX-39. The answer is B. (Chap. 384) NSAIDs are the first line of pharmacologic therapy for ankylosing
spondylitis, for which this patient has a classic presentation. These agents have been shown to
reduce pain and tenderness and increase mobility. There is even some evidence that they slow
disease progression. Given their proven efficacy, tolerability, and safety, they remain first-line
therapy. Anti–TNF-α agents have been reported to have dramatic effects in ankylosing
spondylitis, with infliximab, etanercept, adalimumab, or golimumab having published reports of
success. Because of their potential side effects, including serious infections, hypersensitivity
reactions, and others, these agents should be reserved for patients failing therapy with NSAIDs.

IX-40. The answer is E. (Chaps. 384 and 191) Reactive arthritis refers to an acute, nonpurulent
arthritis that occurs after an infection elsewhere in the body. Often presenting with lower joint
inflammatory arthritis occurring 1–4 weeks after a diarrheal episode, reactive arthritis may also
include uveitis or conjunctivitis, dactylitis, urogenital lesions, and characteristic mucocutaneous
lesions such as keratoderma blennorrhagicum. The most common organisms associated with
reactive arthritis are Shigella species, although Yersinia, Chlamydia, and, to a much lesser extent,
Salmonella and Campylobacter have been described. Although more common in children residing
in developing countries, over 400,000 annual cases of Shigella species infections occur in the
United States. These infections mostly occur in children 4–11 years old. Most U.S. cases are due
to Shigella sonnei, with Shigella flexneri the second most common cause.

IX-41. The answer is E. (Chap. 384) Whipple disease is a rare chronic bacterial infection of the
gastrointestinal tract most commonly affecting middle-aged men. Arthritis is a common early
manifestation of the disease, with arthritis predating gastrointestinal symptoms by 5 years or
more. Large and small joints may be affected, and sacroiliitis is common. Arthritis is often
migratory and lasts several days with spontaneous recovery. Synovial fluid is generally
inflammatory, including polymorphonuclear cells. Radiographs rarely show joint erosions,
although sacroiliitis may be demonstrated. Diagnosis is often made by polymerase chain reaction
amplification of genetic material from Tropheryma whipplei in biopsied material, most commonly
the gut.

IX-42. The answer is C. (Chap. 384) Enthesopathy or enthesitis is the term used to describe
inflammation at the site of tendinous or ligamentous insertion into bone. This type of
inflammation is seen most frequently in patients with seronegative spondyloarthropathies and
various infections, especially viral infections. The other definitions apply to other terms used in
the orthopedic and rheumatic examination. Subluxation is the alteration of joint alignment so
that articulating surfaces incompletely approximate each other. Synovitis refers to inflammation
at the site of tendinous or ligamentous insertion into bone. Inflammation of a saclike cavity near
a joint that decreases friction is the definition of bursitis. Finally, crepitus is a palpable vibratory
or crackling sensation elicited with joint motion.

IX-43. The answer is D. (Chap. 384) Nail changes in the fingers or toes occur in up to 90% of patients
with psoriatic arthritis (PsA), compared with 40% of psoriatic patients without arthritis, and
pustular psoriasis is said to be associated with more severe arthritis. Several articular features
distinguish PsA from other joint disorders; such hallmark features include dactylitis and
enthesitis. Dactylitis occurs in >30%; enthesitis and tenosynovitis are also common and are
probably present in most patients, although often not appreciated on physical examination.
Shortening of digits because of underlying osteolysis is particularly characteristic of PsA, and
there is a much greater tendency than in RA for both fibrous and bony ankylosis of small joints.
Rapid ankylosis of one or more proximal interphalangeal (PIP) joints early in the course of
disease is not uncommon. Back and neck pain and stiffness are also common in PsA. Diarrhea is
not a feature of PsA. See Figure IX-43.
FIGURE IX-43

IX-44. The answer is A. (Chap. 384) Aortic insufficiency, sometimes leading to congestive heart
failure, occurs in a small percentage of patients, occasionally early. Third-degree heart block
may occur alone or together with aortic insufficiency. Subclinical pulmonary lesions and cardiac
dysfunction may be relatively common.

IX-45. The answer is B. (Chap. 385) Although the molecular pathology of most vasculitic syndromes is
poorly understood, the deposition of immune complexes is commonly thought to play an
important role in vasculitis associated with Henoch-Schönlein purpura, cryoglobulinemic
vasculitis associated with hepatitis C, serum sickness and cutaneous vasculitic syndromes, and
polyarteritis nodosa–like vasculitis associated with hepatitis B. Granulomatosis with polyangiitis
(previously Wegener granulomatosis), Churg-Strauss syndrome, and microscopic polyangiitis are
thought to be due to production of antineutrophilic antibodies. Pathogenic T-lymphocyte
responses are also implicated in granulomatosis with polyangiitis (previously Wegener), giant-
cell arteritis, Takayasu arteritis, and Churg-Strauss syndrome.

IX-46. The answer is D. (Chap. 385) The patient presents with classic symptoms for granulomatosis
with polyangiitis. The average age of diagnosis is 40 years, and there is a male predominance.
Upper respiratory symptoms often predate lung or renal findings and may even present with
septal perforation. The diagnosis is made by demonstration of necrotizing granulomatous
vasculitis on biopsy. Pulmonary tissue offers the highest yield. Biopsy of the upper airway
usually shows the granulomatous inflammation but infrequently shows vasculitis. Renal biopsy
may show the presence of pauci-immune glomerulonephritis.

IX-47. The answer is C. (Chap. 385) The patient has a classic presentation for giant-cell arteritis with
associated polymyalgia rheumatica including headache, jaw claudication, and visual
disturbances. Her age makes this diagnosis highly likely as well. The diagnosis is confirmed by
temporal artery biopsy; however, in the presence of visual symptoms, initiation of therapy
should not be delayed pending a biopsy because the biopsy may be positive even after
approximately 14 days of glucocorticoid therapy. Delay in therapy risks irreversible visual loss.
Additionally, a dramatic response to therapy may lend further support to the diagnosis. The
primary therapy is prednisone at 40–60 mg daily for 1 month with gradual tapering. Although
ESR is nearly universally elevated, it is not specific for the diagnosis. Temporal artery ultrasound
may be suggestive but is not diagnostic.

IX-48. The answer is C. (Chap. 385) The most common manifestations of cryoglobulinemic vasculitis
are cutaneous vasculitis, arthritis, peripheral neuropathy, and glomerulonephritis. The
demonstration of circulating cryoprecipitates is a critical component of the diagnosis, and often
rheumatoid factor can be found as well. Because hepatitis C infection is present in the vast
majority of patients with cryoglobulinemic vasculitis, infection should be sought in all patients
with this clinical syndrome.

IX-49. The answer is C. (Chap. 385) The most likely cause of the acute coronary syndrome in this
patient is thrombosis of a coronary artery aneurysm in an individual with a past history of
Kawasaki disease. Kawasaki disease is an acute multisystem disease that primarily presents in
children <5 years of age. The clinical manifestations in childhood are nonsuppurative cervical
lymphadenitis; desquamation of the fingertips; and erythema of the oral cavity, lips, and palms.
Approximately 25% of cases are associated with coronary artery aneurysms that occur late in
illness in the convalescent stage. Early treatment (within 7–10 days of onset) with IV
immunoglobulin and high-dose aspirin decreases the risk of developing coronary aneurysms to
about 5%. Even if coronary artery aneurysms develop, most regress over the course of the first
year if the size is <6 mm. Aneurysms >8 mm, however, are unlikely to regress. Complications
of persistent coronary artery aneurysms include rupture, thrombosis and recanalization, and
stenosis at the outflow area. Dissection of the aortic root and coronary ostia is a common cause
of death in Marfan syndrome and can also be seen with aortitis due to Takayasu arteritis. In this
patient, there is no history of hypertension, limb ischemia, or systemic symptoms that would
suggest an active vasculitis. In addition, there are no other ischemic symptoms that would be
expected in Takayasu arteritis. Myocardial bridging overlying a coronary artery is seen
frequently at autopsy but is an unusual cause of ischemia. The possibility of cocaine use as a
cause of myocardial ischemia in a young individual must be considered, but given the clinical
history, it is a less likely cause of ischemia in this case.

IX-50. The answer is E. (Chap. 385) Prior to initiation of azathioprine, thiopurine methyltransferase
(TPMT), an enzyme involved in the metabolism of azathioprine, should be assayed because
inadequate levels may result in severe cytopenia. The antineutrophil cytoplasmic antibody
(ANCA) titer can be misleading and should not be used to assess disease activity. Many patients
who achieve remission continue to have elevated titers for years. Results from a large
prospective study found that increases in ANCA were not associated with relapse and that only
43% of patients relapsed within 1 year of an increase in ANCA levels. Thus, a rise in ANCA by
itself is not a harbinger of immediate disease relapse and should not lead to reinstitution or
increase in immunosuppressive therapy.

IX-51. The answer is C. (Chap. 385) Polyarteritis nodosa does not involve pulmonary arteries,
although bronchial vessels may be involved; granulomas, significant eosinophilia, and an allergic
diathesis are not observed. Vascular beds typically involved are listed in Table IX-51.

TALBLE IX-51 Clinical Manifestations Related to Organ System Involvement in Polyarteritis


Nodosa
IX-52. The answer is C. (Chap. 386e) Granulomatosis with polyangiitis (Wegener) is a small-vessel
vasculitis that involves the lung in >80% of cases. One-third of patients with radiographic
abnormalities may be asymptomatic. It typically also involves the upper respiratory tract and the
kidney. Surgical biopsies of radiographically abnormal pulmonary parenchyma have a diagnostic
yield of approximately 90% in patients with granulomatosis with polyangiitis (Wegener). Biopsy
may also differentiate vasculitis from infection or malignancy. The yield of bronchoscopic
transbronchial biopsy is substantially lower than surgical biopsy. Cryoglobulinemic vasculitis
and IgA vasculitis (Henoch-Schönlein) are small-vessel vasculitides that typically involve the skin
and kidney. Polyarteritis nodosa is a medium-vessel vasculitis that typically involves the
mesenteric vessels. Cutaneous vasculitis represents the most common vasculitic feature and can
be seen in a broad spectrum of settings including infections, medications, malignancies, and
connective tissue diseases.

IX-53. The answer is A. (Chap. 386e) In granulomatosis with polyangiitis (Wegener), 80% of patients
may have pulmonary involvement during their disease course. The typical chest radiographic
abnormalities are single or multiple nodular infiltrates that often cavitate (Figures IX-53A and
IX-53B).
FIGURE IX-53A

FIGURE IX-53B
FIGURE IX-53C

However, due to the frequent involvement of airways, tracheal or bronchial stenosis may occur.
Ground glass infiltrates (Figure IX-53C) may occur due to capillaritis and subsequent pulmonary
hemorrhage.
Microscopic polyangiitis and SLE are also causes of capillaritis. Patients with granulomatosis with
polyangiitis on immunosuppressive therapy are at risk of opportunistic lung infections.
Granulomatosis with polyangiitis typically will not cause bronchiectasis in the absence of a
history of multiple respiratory infections. Bronchiectasis is more typical of cystic fibrosis, ciliary
dysfunction syndromes, obstructive lung disease, or congenital immunodeficiency.

IX-54. The answer is C. (Chap. 387) Recurrent oral ulceration is required for the diagnosis of Behçet
disease. The ulcers may be single or multiple, are shallow based with a yellow necrotic base, and
are painful. They are generally small, <10 mm in diameter. In addition, the diagnosis of Behçet
disease requires two of the following: recurrent genital ulceration, eye lesions, skin lesions, and
pathergy test. Nonspecific skin inflammatory reactivity to any scratches or intradermal saline
injection (pathergy test) is common and specific.

IX-55. The answer is A. (Chap. 387) Behçet’s disease is a multisystem disorder of uncertain cause that
is marked by oral and genital ulcerations and ocular involvement. This disorder affects males
and females equally and is more common in persons of Mediterranean, Middle Eastern, and Far
Eastern descent. Approximately 50% of these persons have circulating autoantibodies to human
oral mucosa. The clinical features are quite varied. The presence of recurrent aphthous
ulcerations is essential for the diagnosis. Most of these patients have primarily oral ulcerations,
although genital ulcerations are more specific for the diagnosis. The ulcers are generally painful,
can be shallow or deep, and last for 1 or 2 weeks. Other skin involvement may occur, including
folliculitis, erythema nodosum, and vasculitis. Eye involvement is the most dreaded complication
because it may progress rapidly to blindness. It often presents as panuveitis, iritis, retinal vessel
occlusion, or optic neuritis. This patient also presents with superficial venous thrombosis.
Superficial and deep venous thromboses are present in one-fourth of these patients. Neurologic
involvement occurs in up to 10%. Laboratory findings are nonspecific with elevations in the ESR
and the white blood cell count. Bullous pemphigoid is a polymorphic autoimmune subepidermal
blistering disease usually seen in the elderly. Initial lesions may consist of urticarial plaques;
most patients eventually display tense blisters on either normal-appearing or erythematous skin.
The lesions are usually distributed over the lower abdomen, groin, and flexor surface of the
extremities; oral mucosal lesions are found in some patients. Discoid lupus erythematosus is the
cutaneous form of SLE and is characterized by atrophic, depigmented plaques and patches
surrounded by hyperpigmentation and erythema in association with scarring and alopecia.
IX-56. The answer is E. (Chap. 387) Superficial or deep peripheral vein thrombosis is seen in 30% of
patients. Pulmonary emboli are a rare complication. The superior vena cava is obstructed
occasionally, producing a dramatic clinical picture. Arterial involvement occurs in less than 5%
of patients and presents with aortitis or peripheral arterial aneurysm and arterial thrombosis.
Pulmonary artery vasculitis presenting with dyspnea, cough, chest pain, hemoptysis, and
infiltrates on chest roentgenograms has been reported in 5% of patients and should be
differentiated from thromboembolic disease because it warrants anti-inflammatory and not
thrombolytic therapy. Neurologic involvement (5%–10%) appears mainly in the parenchymal
form (80%); it is associated with brainstem involvement and has a serious prognosis (central
nervous system [CNS]-Behçet disease). IL-6 is persistently raised in cerebrospinal fluid of these
patients. Cerebral venous thrombosis is most frequently observed in the superior sagittal and
transverse sinuses and is associated with headache and increased intracranial pressure. MRI
and/or proton magnetic resonance spectroscopy are very sensitive and should be employed if
CNS-Behçet disease is suspected. Gastrointestinal involvement is seen more frequently in patients
from Japan and consists of mucosal ulcerations of the gut, resembling Crohn disease.
Epididymitis is seen in 5% of patients, whereas amyloidosis of AA type and glomerulonephritis
are uncommon.

IX-57. The answer is B. (Chap. 388) When patients present with proximal muscle weakness and
myositis, whether polymyositis, dermatomyositis, or inclusion body myositis, the diagnosis is
confirmed by analysis of serum muscle enzymes, electromyography (EMG) findings, and muscle
biopsy. The most sensitive serum enzyme is creatinine kinase (CK), which can be elevated as
much as 50-fold in active disease. CK levels usually parallel disease activity, but can be normal
in some patients with inclusion body myositis or dermatomyositis. CK is always elevated in
active polymyositis and thus is considered most sensitive. Other enzymes may be elevated as
well, including glutamic-oxaloacetic transaminase, glutamate pyruvate transaminase, lactate
dehydrogenase, and aldolase.

IX-58. The answer is C. (Chap. 388) Various autoantibodies against nuclear antigens (e.g.m ANAs) and
cytoplasmic antigens are found in up to 20% of patients with inflammatory myopathies. The
antibodies to cytoplasmic antigens are directed against ribonucleoproteins involved in protein
synthesis (antisynthetases) or translational transport (anti–signal-recognition particles). The
antibody directed against the histidyl-transfer RNA synthetase, called anti-Jo-1, accounts for
75% of all the antisynthetases and is clinically useful because up to 80% of patients with this
autoantibody will have interstitial lung disease. Patients with anti-Jo-1 may also have Raynaud
phenomenon, nonerosive arthritis, and the MHC molecules DR3 and DRw52. Interstitial lung
disease associated with anti-Jo-1 is often rapidly progressive and fatal, even if treated
aggressively with cyclophosphamide or other immunosuppressants.

IX-59. The answer is A. (Chap. 388) Dermatomyositis is associated with malignancy in up to 15% of
cases; thus, age-appropriate cancer screening is indicated when this diagnosis is made.
Exhaustive cancer searches are not recommended, however. Dermatomyositis may be associated
occasionally with scleroderma and mixed connective tissue disease, but less frequently with SLE,
RA, or Sjögren syndrome, which are more closely associated with polymyositis or inclusion body
myositis (IBM). Viruses may be associated with IBM and polymyositis but are not proven to be
associated with dermatomyositis. Parasites and bacteria such as cestodes and nematodes are
associated with polymyositis, but not other forms of inflammatory myopathy. Finally, thyroid-
stimulating immunoglobulins are not known to be associated with dermatomyositis.

IX-60. The answer is A. (Chap. 388) Mild statin-induced myopathy is noninflammatory and usually
resolves with discontinuation of therapy. In rare patients, however, muscle weakness continues
to progress even after the statin is withdrawn; in these cases, a diagnostic muscle biopsy is
indicated and search for antibodies to 3-hydroxy-3-methylglutarylcoenzyme A reductase
(HMGCR) is suggested; if histologic evidence of polymyositis or necrotizing myositis is present,
immunotherapy should be initiated. ANA may be present but is nonspecific in these cases. Anti-
Jo-1 antibodies are associated with the antisynthetase syndromes where myositis is usually
accompanied by interstitial lung disease and stereotypical skin changes. Antibodies against signal
recognition particle (SRP) are not associated with statin-induced myopathy.

IX-61. The answer is E. (Chap. 389) Relapsing polychondritis most often presents with recurrent
painful swelling of the ear. Although other cartilaginous sites may be involved such as the nose
and the tracheobronchial tree, these are less frequent. Episodes of ear involvement may result in
floppy ears. Typically the pinna is affected while the earlobe is spared because there is no
cartilage in the lobe. Cogan syndrome is a rare vasculitic syndrome involving hearing loss, but
cartilage inflammation is not a feature. Recurrent trauma or irritation is a consideration, but the
history is not suggestive, and it would less likely be bilateral and accompanied by inflammatory
findings and a relatively spared earlobe.

IX-62. The answer is E. (Chap. 390) Despite multiple investigations, the cause of sarcoidosis remains
unknown. Currently, the most likely etiology is an infectious or noninfectious environmental
agent that triggers an inflammatory response in a genetically susceptible host. Among the
possible infectious agents, careful studies have shown a much higher incidence of
Propionibacterium acnes in the lymph nodes of sarcoidosis patients compared to controls. An
animal model has shown that P acnes can induce a granulomatous response in mice similar to
sarcoidosis. Others have demonstrated the presence of a mycobacterial protein (Mycobacterium
tuberculosis catalase-peroxidase [mKatG]) in the granulomas of some sarcoidosis patients. This
protein is very resistant to degradation and may represent the persistent antigen in sarcoidosis.
Immune response to this and other mycobacterial proteins has been documented by another
laboratory. These studies suggest that a Mycobacterium similar to M tuberculosis could be
responsible for sarcoidosis. The mechanism of exposure/infection with such agents has been the
focus of other studies. Environmental exposures to insecticides and mold have been associated
with an increased risk for disease. In addition, healthcare workers appear to have an increased
risk. Also, sarcoidosis in a donor organ has occurred after transplantation into a sarcoidosis
patient. Some authors have suggested that sarcoidosis is not due to a single agent but represents
a particular host response to multiple agents. Some studies have been able to correlate the
environmental exposures to genetic markers. These studies have supported the hypothesis that a
genetically susceptible host is a key factor in the disease. Although helper T cells may be
increased, particularly in the lung of patients with sarcoidosis, it is not a monoclonal or
malignant expansion of cells.

IX-63. The answer is D. (Chap. 390) Lung involvement occurs in >90% of sarcoidosis patients and is
by far the most common manifestation of sarcoidosis. Characteristic computed tomography (CT)
features include peribronchial thickening and reticular nodular changes, which are
predominantly subpleural. The peribronchial thickening seen on CT scan seems to explain the
high yield of granulomas from bronchial biopsies performed for diagnosis. Usually the infiltrates
in sarcoidosis are predominantly an upper lobe process. Approximately one-half of sarcoidosis
patients present with obstructive disease, reflected by a reduced ratio of forced expiratory
volume in 1 second to forced vital capacity. Cough is a common symptom. Airway involvement
causing varying degrees of obstruction underlies the cough in most sarcoidosis patients.
Pulmonary arterial hypertension is reported in at least 5% of sarcoidosis patients. Either direct
vascular involvement or the consequence of fibrotic changes in the lung can lead to pulmonary
arterial hypertension. In sarcoidosis patients with end-stage fibrosis awaiting lung transplant,
70% will have pulmonary arterial hypertension. This is a much higher incidence than that
reported for other fibrotic lung diseases. In less advanced, but still symptomatic, patients,
pulmonary arterial hypertension has been noted in up to 50% of the cases. Because sarcoidosis-
associated pulmonary arterial hypertension may respond to therapy, evaluation for this should
be considered in persistently dyspneic patients.

IX-64. The answer is D. (Chap. 390) Hypercalcemia and/or hypercalciuria occurs in about 10% of
sarcoidosis patients. It is more common in whites than African Americans and in men. The
mechanism of abnormal calcium metabolism is increased production of 1,25-dihydroxyvitamin D
by the granuloma itself. The 1,25-dihydroxyvitamin D causes increased intestinal absorption of
calcium, leading to hypercalcemia with a suppressed parathyroid hormone (PTH) level.
Increased exogenous vitamin D from diet or sunlight exposure may exacerbate this problem.
Serum calcium should be determined as part of the initial evaluation of all sarcoidosis patients,
and a repeat determination may be useful during the summer months with increased sun
exposure.

IX-65. The answer is C. (Chap. 390) The presence of cardiac involvement is influenced by race.
Although over a quarter of Japanese sarcoidosis patients develop cardiac disease, only 5% of
sarcoidosis patients in the United States and Europe develop symptomatic cardiac disease.
However, there is no apparent racial predilection between whites and African Americans.

IX-66. The answer is E. (Chap. 390) Cardiac sarcoidosis classically presents as either congestive heart
failure or cardiac arrhythmias, which results from infiltration of the heart muscle by granulomas.
Valvular, coronary, or pericardial involvement with granulomatous disease has also been
described. Diffuse granulomatous involvement of the heart muscle can lead to profound
dysfunction with left ventricular ejection fractions below 10%. Even in this situation,
improvement in the ejection fraction can occur with systemic therapy. Arrhythmias can also
occur with diffuse infiltration or with more patchy cardiac involvement. If the atrioventricular
node is infiltrated, heart block can occur, which can be detected by routine electrocardiography.
Ventricular arrhythmias and sudden death due to ventricular tachycardia are common causes of
death. Arrhythmias are best detected using 24-hour ambulatory monitoring, and
electrophysiology studies may be negative. Other screening tests for cardiac disease include
routine electrocardiography and echocardiography. The confirmation of cardiac sarcoidosis is
usually performed with either MRI or positron emission tomography (PET) scanning.

IX-67. The answer is D. (Chap. 391e) This patient has IgG4-related disease (IgG4-RD). IgG4-RD is a
fibroinflammatory condition characterized by a tendency to form tumefactive lesions. The
clinical manifestations of this disease, however, are protean and continue to be defined.
Pancreatic and retroperitoneal involvement is well described and may present as type 1
autoimmune pancreatitis, presenting as mild abdominal pain, weight loss, and acute, obstructive
jaundice, mimicking adenocarcinoma of the pancreas (including a pancreatic mass). Imaging
shows diffuse (termed sausage-shaped pancreas) or segmental pancreatic enlargement, with loss of
normal lobularity; a mass often raises the suspicion of malignancy. The key histopathology
characteristics of IgG4-RD are a dense lymphoplasmacytic infiltrate that is organized in a
storiform pattern (resembling a basket weave), obliterative phlebitis, and a mild to moderate
eosinophilic infiltrate. The inflammatory infiltrate is composed of an admixture of B and T
lymphocytes. B cells are typically organized in germinal centers. Plasma cells staining for CD19,
CD138, and IgG4 appear to radiate out from the germinal centers. Vital organ involvement must
be treated aggressively, however, because IgG4-RD can lead to serious organ dysfunction and
failure. Aggressive disease can lead quickly to end-stage liver disease, permanent impairment of
pancreatic function, renal atrophy, aortic dissection or aneurysms, and destructive lesions in the
sinuses and nasopharynx. Glucocorticoids are the first line of therapy. Treatment regimens,
extrapolated from experience with the management of autoimmune pancreatitis, generally begin
with 40 mg/d of prednisone, with tapering to discontinuation or maintenance doses of 5 mg/d
within 2 or 3 months. The clinical response to glucocorticoids is usually swift and striking;
however, longitudinal data indicate that disease flares occur in more than 90% of patients within
3 years. Conventional steroid-sparing agents such as azathioprine and mycophenolate mofetil
have been used in some patients; however, evidence for their efficacy is lacking.

IX-68. The answer is A. (Chap. 392) Familial Mediterranean fever (FMF) is the prototype of a group of
inherited diseases that are characterized by recurrent episodes of fever with serosal, synovial, or
cutaneous inflammation. The FMF gene encodes a 781-amino acid, ~95-kDa protein denoted
pyrin (or marenostrin) that is expressed in granulocytes, eosinophils, monocytes, dendritic cells,
and synovial and peritoneal fibroblasts. Typical FMF episodes generally last 24–72 hours, with
arthritic attacks tending to last somewhat longer. In some patients, the episodes occur with great
regularity, but more often, the frequency of attacks varies over time, ranging from as often as
once every few days to remissions lasting several years. Attacks are often unpredictable,
although some patients relate them to physical exertion, emotional stress, or menses; pregnancy
may be associated with remission.

If measured, fever is nearly always present throughout FMF attacks. Severe hyperpyrexia and
even febrile seizures may be seen in infants, and fever is sometimes the only manifestation of
FMF in young children. Over 90% of FMF patients experience abdominal attacks at some time.
Episodes range in severity from dull, aching pain and distention with mild tenderness on direct
palpation to severe generalized pain with absent bowel sounds, rigidity, rebound tenderness, and
air-fluid levels on upright radiographs. CT scanning may demonstrate a small amount of fluid in
the abdominal cavity. If such patients undergo exploratory laparotomy, a sterile, neutrophil-rich
peritoneal exudate is present, sometimes with adhesions from previous episodes. Ascites is rare.
Exercise-induced (nonfebrile) myalgia is common in FMF, and a small percentage of patients
develop a protracted febrile myalgia that can last several weeks.

IX-69. The answer is A. (Chap. 392) The treatment of choice for FMF is daily oral colchicine, which
decreases the frequency and intensity of attacks and prevents the development of amyloidosis in
compliant patients. Intermittent dosing at the onset of attacks is not as effective as daily
prophylaxis and is of unproven value in preventing amyloidosis. The usual adult dose of
colchicine is 1.2–1.8 mg/d, which causes substantial reduction in symptoms in two-thirds of
patients and some improvement in >90%. Children may require lower doses, although not
proportionately to body weight.

IX-70. The answer is E. (Chap. 394) Osteoarthritis (OA) affects certain joints, yet spares others.
Commonly, affected joints include the cervical and lumbosacral spine, hip, knee, and first
metatarsal phalangeal joint (MTP). In the hands, the distal and proximal interphalangeal joints
and the base of the thumb are often affected. Usually spared are the wrist, elbow, and ankle.
Human joints were designed, in an evolutionary sense, for brachiating apes, animals that still
walked on four limbs. We thus develop OA in joints that were ill designed for human tasks such
as pincer grip (OA in the thumb base) and walking upright (OA in knees and hips). Some joints,
like the ankles, may be spared because their articular cartilage may be uniquely resistant to
loading stresses. See Figure IX-70.
FIGURE IX-70

IX-71. The answer is D. (Chap. 394) Examination of the synovial fluid is often helpful in suspected
osteoarthritis, particularly if inflammatory disease is possible. If the synovial fluid white blood
cell count is >1000/μL, inflammatory arthritis, gout, or pseudogout is likely, with the latter two
being also identified by the presence of crystals. Because cartilage is aneural, cartilage loss in a
joint is not accompanied by pain. Thus, pain in OA likely arises from structures outside the
cartilage. Innervated structures in the joint include the synovium, ligaments, joint capsule,
muscles, and subchondral bone. Most of these are not visualized by the x-ray, and the severity of
radiographic changes in OA correlates poorly with pain severity. OA is the most common type of
arthritis. Its high prevalence, especially in the elderly, and the high rate of disability related to
disease make it a leading cause of disability in the elderly. Although MRI may reveal the extent
of pathology in an osteoarthritic joint, it is not indicated as part of the diagnostic workup.

IX-72. The answer is C. (Chap. 394) Recent guidelines recommend against the use of glucosamine or
chondroitin for OA. Large publicly supported trials have failed to show that, compared with
placebo, these compounds relieve pain in persons with disease. Glucocorticoid injections are
efficacious in OA, but response is variable, with some patients having little relief of pain,
whereas others experience pain relief lasting several months. Glucocorticoid injections are useful
to get patients over acute flares of pain and may be especially indicated if the patient has
coexistent OA and crystal deposition disease, especially from calcium pyrophosphate dihydrate
crystals. Acetaminophen (paracetamol) is the initial analgesic of choice for patients with OA in
knees, hips, or hands. For some patients, it is adequate to control symptoms, in which case more
toxic drugs such as NSAIDs can be avoided. Doses up to 1 g three times daily can be used.
NSAIDs are the most popular drugs to treat osteoarthritic pain. They can be administered either
topically or orally. In clinical trials, oral NSAIDs produced approximately 30% greater
improvement in pain than high-dose acetaminophen. Ultimately, when the patient with knee or
hip OA has failed medical treatment modalities and remains in pain, with limitations of physical
function that compromise the quality of life, the patient should be referred for total knee or hip
arthroplasty. These are highly efficacious operations that relieve pain and improve function in
the vast majority of patients, although rates of success are higher for hip than knee replacement.

IX-73. The answer is E. (Chap. 395) Figure IX-73B illustrates extracellular and intracellular
monosodium urate crystals, as seen in a fresh preparation of synovial fluid. This is gout. Most
patients with gout are overproducers of uric acid. Hyaline cartilage degeneration is typical of
osteoarthritis, which usually has a bland synovial aspirate. Antibodies to ANA are typical of
lupus, rare in an elderly man, and not associated with crystalline fluid. Bacterial joint infection
would have a purulent synovial fluid. Increased production of inorganic pyrophosphate is a
cause of calcium pyrophosphate deposition disease (CPPD; pseudogout), another crystalline
arthropathy. CPPD crystals are shown in Figure IX-73B.

FIGURE IX-73B

IX-74. The answer is C. (Chap. 395) The xanthine oxidase inhibitor allopurinol is by far the most
commonly used hypouricemic agent and is the best drug to lower serum urate in overproducers,
urate stone formers, and patients with renal disease. It can be given in a single morning dose,
usually 100 mg initially and increasing up to 800 mg if needed. In patients with chronic renal
disease, the initial allopurinol dose should be lower and adjusted depending on the serum
creatinine concentration; for example, with a creatinine clearance of 10 mL/min, one generally
would use 100 mg every other day. Doses can be increased gradually to reach the target urate
level of <6 mg/dL. Toxicity of allopurinol has been recognized increasingly in patients who use
thiazide diuretics, in patients allergic to penicillin and ampicillin, and in Asians expressing HLA-
B*5801. Colchicine is commonly used with allopurinol in the treatment of gout. Allopurinol and
azathioprine should not be coprescribed because azathioprine can greatly increase blood levels
of allopurinol and lead to toxicity.

IX-75. The answer is C. (Chap. 396) This patient presents with a characteristic history for
fibromyalgia, a diffuse pain syndrome associated with increased sensitivity to evoked pain. The
underlying pathophysiology of pain in fibromyalgia is felt to be related to altered pain
processing in the central nervous system. Epidemiologically, women are affected nine times more
frequently than men. The worldwide prevalence of fibromyalgia is 2%–3%, but in primary care
practices, it is as high as 5%–10%. The disorder is even more common in patients with
degenerative or inflammatory rheumatic disorders, with a prevalence of 20% or higher. The
most common presenting complaint is diffuse pain that is difficult to localize. Pain is both above
and below the waist and affects the extremities as well as the axial skeleton. However, it does
not localize to a specific joint. The pain is noted to be severe in intensity and difficult to ignore
and interferes with daily functioning. Although this patient demonstrates pain at several tender
points, the American College of Rheumatology no longer includes tender point assessment in the
diagnostic criteria for fibromyalgia. Rather, the new criteria focus on clinical symptoms of
widespread pain and neuropsychological symptoms that have been present for at least 3 months.
Some of the neuropsychological conditions that are frequently observed in fibromyalgia include
sleep disturbance, impaired cognitive functioning, fatigue, stiffness, anxiety, and depression. The
lifetime prevalence of mood disorders in patients with fibromyalgia is 80%. Sleep disturbances
can include difficulty falling asleep, difficulty staying asleep, or nonrestorative sleep, among
others.

IX-76. The answer is D. (Chap. 396) Fibromyalgia is a common disorder affecting 2%–5% of the
population. It presents as a diffuse pain syndrome with associated neuropsychological symptoms
including depression, anxiety, fatigue, cognitive dysfunction, and disturbed sleep. Treatment for
fibromyalgia should include a combination of nonpharmacologic and pharmacologic approaches.
Patient education regarding the disease is important to provide a framework for understanding
symptoms. The focus of treatment should not be on eliminating pain, but rather improving
function and quality of life. Physical conditioning is an important part of improving function and
should include a multifaceted exercise program with aerobic exercise, strength training, and
exercises that incorporate relaxation techniques such as yoga or tai chi. Cognitive behavioral
therapy can be useful in improving sleep disturbance and also for decreasing illness behaviors.
Pharmacologic therapy in fibromyalgia is targeted at the afferent and efferent pain pathways.
The two most common categories of medications for fibromyalgia are antidepressants and
anticonvulsants. Amitriptyline, duloxetine, and milnacipran have all been used with some
efficacy in fibromyalgia. Duloxetine and milnacipran are approved by the U.S. Food and Drug
Administration (FDA) for the treatment of fibromyalgia. The anticonvulsants that are
predominantly used in fibromyalgia are those that are ligands of the α-2-δ subunit of voltage-
gated calcium channels. These include gabapentin and pregabalin, which is also FDA approved
for treatment of fibromyalgia. Anti-inflammatory medications and glucocorticoids are not
effective in fibromyalgia. However, if there is a comorbid triggering condition such as RA,
appropriate therapy directed at the underlying disorder is critical to controlling symptoms of
fibromyalgia as well. Opioid analgesics such as oxycodone should be avoided. They have no
efficacy in treating fibromyalgia and may induce hyperalgesia that can worsen both pain and
function.

IX-77. The answer is A. (Chap. 396) Fibromyalgia is characterized by chronic widespread


musculoskeletal pain, stiffness, paresthesia, disturbed sleep, and easy fatigability. It occurs in a
9:1 female-to-male ratio. It is not confined to any particular region, ethnicity, or climate.
Although the pathogenesis is not clear, there are associations with disturbed sleep and abnormal
pain perception. Fibromyalgia is diagnosed by the presence of widespread pain, a history of
widespread musculoskeletal pain that has been present for >3 months, and presence of
neuropsychological dysfunction (fatigue, waking unrefreshed, or cognitive symptoms). In the
prior diagnostic criteria, it was required to demonstrate pain on palpation at 11 of 18 tender
point sites. However, this was abandoned in the updated criteria because it was felt that strict
application of a threshold of pain could lead to underdiagnosis of the disorder. Besides pain on
palpation, the neurologic and musculoskeletal examinations are normal in patients with
fibromyalgia. Psychiatric illnesses, particularly depression and anxiety disorders, are common
comorbidities in these patients but do not help satisfy any diagnostic criteria.

IX-78. The answer is A. (Chap. 397) The finding shown in Figure IX-78 is characteristic of clubbing.
Clubbing occurs in the distal portions of the digits and is characterized by widening of the
fingertips, convexity of the nail contour, and loss of the normal 15-degree angle between the
proximal nail and cuticle. Clinically, it can be sometimes difficult to ascertain whether clubbing
is present. One approach to the diagnosis of clubbing is to measure the diameter of the finger at
the base of the nail and at the tip of the finger in all 10 fingers. For each finger, a ratio between
the base of the nail and the tip of the finger is determined. If the sum of all 10 fingers is greater
than 1, then clubbing is felt to be present. A simpler approach is to have an individual place the
dorsal surfaces of the distal fourth digits from each hand together. In a normal individual, there
should be a diamond-shaped space between the digits. When an individual has clubbing, this
space is obliterated. Clubbing most commonly occurs in advanced lung disease, especially
bronchiectasis, cystic fibrosis, and interstitial lung diseases like sarcoidosis or idiopathic
pulmonary fibrosis. Clubbing was originally described in individuals with empyema and can
occur in chronic lung infections, including lung abscess, tuberculosis, or fungal infections.
Pulmonary vascular lesions and lung cancer also are associated with clubbing. However, chronic
obstructive pulmonary disease does not cause clubbing. However, the causes of clubbing are not
limited to the pulmonary system alone. Clubbing can be a benign familial condition and is also
associated with a variety of other disorders, including cyanotic congenital heart disease,
subacute bacterial endocarditis, Crohn disease, ulcerative colitis, celiac disease, and cancer of the
esophagus, liver, small bowel, and large bowel. In untreated hyperthyroidism, clubbing can
occur in association with periostitis in a condition called thyroid acropachy. Although these
numerous clinical associations have been described for many centuries, the cause of clubbing
remains unknown.

IX-79. The answer is C. (Chap. 397) Symptoms of hemochromatosis usually begin between the ages of
40 and 60 but can appear earlier. Arthropathy, which occurs in 20%–40% of patients, usually
begins after the age of 50 and may be the first clinical feature of hemochromatosis. The
arthropathy is an osteoarthritis-like disorder affecting the small joints of the hands and later the
larger joints, such as knees, ankles, shoulders, and hips. The second and third
metacarpophalangeal joints of both hands are often the first and most prominent joints affected;
this clinical picture may provide an important clue to the possibility of hemochromatosis
because these joints are not predominantly affected by “routine” osteoarthritis. Patients
experience some morning stiffness and pain with use of involved joints. The affected joints are
enlarged and mildly tender. Radiographs show narrowing of the joint space, subchondral
sclerosis, subchondral cysts, and juxta-articular proliferation of bone. Hooklike osteophytes are
seen in up to 20% of patients; although they are regarded as a characteristic feature of
hemochromatosis, they can also occur in osteoarthritis and are not disease specific. Bony
erosions are typical of rheumatoid arthritis, not hemochromatosis. The synovial fluid is
noninflammatory. In approximately half of patients, there is evidence of calcium pyrophosphate
deposition disease, and some patients experience episodes of acute pseudogout late in the course
of disease (Chap. 395). The treatment of hemochromatosis is repeated phlebotomy.
Unfortunately, this treatment has little effect on established arthritis, which, along with
chondrocalcinosis, may progress. Symptom-based treatment of the arthritis consists of
administration of acetaminophen and NSAIDs, as tolerated. Acute pseudogout attacks are treated
with high doses of an NSAID or a short course of glucocorticoids. Hip or knee total joint
replacement has been successful in advanced disease.

IX-80. The answer is E. (Chap. 398) Trochanteric bursitis is a common cause of hip pain and results
from inflammation within the bursa that surrounds the insertion of the gluteus medius onto the
greater trochanter of the femur. Bursae lie throughout the body with the purpose of facilitating
movement of tendons and muscles over bony prominences. Bursitis has many causes, including
overuse, trauma, systemic disease, or infection. Trochanteric bursitis typically presents with
acute or subacute hip pain with a varying quality. The pain localizes to the lateral aspect of the
hip and upper thigh. Direct palpation over the posterior aspect of the greater trochanter
reproduces the pain, and often sleeping on the affected side is painful. Pain is also elicited with
external rotation and resisted abduction of the hip. Treatment of trochanteric bursitis includes
use of NSAIDs and avoidance of overuse. If the pain persists, steroid injection into the affected
bursa may be beneficial.

Other causes of hip pain include osteoarthritis, avascular necrosis, meralgia paresthetica,
septic arthritis, occult hip fracture, and referred pain from lumbar spine disease. In patients with
true disorders of the hip joint such as osteoarthritis, avascular necrosis, and occult hip fracture,
the pain is most commonly localized to the groin area. Meralgia paresthetica (lateral femoral
nerve entrapment syndrome) causes a neuropathic pain in the upper outer thigh with symptoms
ranging from tingling sensations to a burning pain. When degenerative spinal disease is the cause
of referred hip pain, there is typically back pain as well. In addition, palpation over the lateral
joint would not reproduce the pain. Iliotibial band syndrome causes lateral knee pain but not hip
pain.

IX-81. The answer is B. (Chap. 398) The iliotibial band is comprised of thick connective tissue that
runs along the outer thigh from the ilium to the fibula. When this band becomes tightened or
inflamed, pain most commonly occurs where the band passes over the lateral femoral condyle of
the knee, leading to a burning or aching pain in this area that can radiate toward the outer
thigh. This overuse injury is most often seen in runners and can be caused by improperly fitted
shoes, running on uneven surfaces, and excessive running. It is also more common in individuals
with a varus alignment of the knee (bowlegged). Treatment of iliotibial band syndrome involves
rest, NSAIDs, physical therapy, and addressing risk factors such as poorly fitted shoes or uneven
running surface. Glucocorticoid injection at the lateral femoral condyle may alleviate pain, but
running must strictly be avoided for 2 weeks following injection. In refractory cases, surgical
release of the iliotibial band may be beneficial.

IX-82. The answer is A. (Chap. 398) Adhesive capsulitis is characterized by pain and restricted motion
of the shoulder. Usually this occurs in the absence of intrinsic shoulder disease, including
osteoarthritis and avascular necrosis. It is, however, more common in patients who have had
bursitis or tendinitis previously as well as patients with other systemic illnesses, such as chronic
pulmonary disease, ischemic heart disease, and diabetes mellitus. The etiology is not clear, but
adhesive capsulitis appears to develop in the setting of prolonged immobility. Reflex sympathetic
dystrophy may also occur in the setting of adhesive capsulitis. Clinically, this disorder is more
commonly seen in females over age 50. Pain and stiffness develop over the course of months to
years. On physical examination, the affected joint is tender to palpation, with a restricted range
of motion. The gold standard for diagnosis is arthrography with limitation of the amount of
injectable contrast to less than 15 mL. In most patients, adhesive capsulitis will regress
spontaneously within 1 to 3 years. NSAIDs, glucocorticoid injections, physical therapy, and early
mobilization of the arm are useful therapies.

IX-83. The answer is B. (Chap. 398) Inflammation of the abductor pollicis longus and the extensor
pollicis brevis at the radial styloid process tendon sheath is known as De Quervain tenosynovitis.
Repetitive twisting of the wrist can lead to this condition. Pain occurs when grasping with the
thumb and can extend radially along the wrist to the radial styloid process. Mothers often
develop this tenosynovitis by holding their babies with the thumb outstretched. The Finkelstein
sign is positive in De Quervain tenosynovitis. It is positive if the patient develops pain by placing
the thumb in the palm, closing the fingers around the thumb, and deviating the wrist in the
ulnar direction. Management of De Quervain tenosynovitis includes NSAIDs and splinting.
Glucocorticoid injections can be effective. A Phalen maneuver is used to diagnose carpal tunnel
syndrome and does not elicit pain. The wrists are flexed for 60 seconds to compress the median
nerve to elicit numbness, burning, or tingling. Gouty arthritis will present with an acutely
inflamed joint with crystal-laden fluid. Rheumatoid arthritis is a systemic illness with
characteristic joint synovitis and radiographic features.

IX-84. The answer is C. (Chap. 398) Ms. Rumpulo has plantar fasciitis, a diagnosis that often can be
made clinically. It is a common cause of foot pain in adults, with the peak incidence occurring in
people between the ages of 40 and 60 years. The pain originates at or near the site of the plantar
fascia attachment to the medial tuberosity of the calcaneus. Several factors that increase the risk
of developing plantar fasciitis include obesity, pes planus (flat foot or absence of the foot arch
when standing), pes cavus (high-arched foot), limited dorsiflexion of the ankle, prolonged
standing, walking on hard surfaces, and faulty shoes. In runners, excessive running and a change
to a harder running surface may precipitate plantar fasciitis. Smoking and oral contraceptives are
not specific risk factors. Patients experience severe pain with the first steps on arising in the
morning or following inactivity during the day. The pain usually lessens with weight-bearing
activity during the day, only to worsen with continued activity. Pain is made worse on walking
barefoot or up stairs. On examination, maximal tenderness is elicited on palpation over the
inferior heel corresponding to the site of attachment of the plantar fascia. Imaging studies may
be indicated when the diagnosis is not clear. Plain radiographs may show heel spurs, which are
of little diagnostic significance. Ultrasonography in plantar fasciitis can demonstrate thickening
of the fascia and diffuse hypoechogenicity, indicating edema at the attachment of the plantar
fascia to the calcaneus. MRI is a sensitive method for detecting plantar fasciitis, but it is usually
not required for establishing the diagnosis. The differential diagnosis of inferior heel pain
includes calcaneal stress fractures, the spondyloarthritides, rheumatoid arthritis, gout, neoplastic
or infiltrative bone processes, and nerve compression/entrapment syndromes.

Resolution of plantar fasciitis symptoms occurs within 12 months in more than 80% of
patients. Initial treatment consists of ice, heat, massage, and stretching. Orthotics providing
medial arch support can be effective. A short course of NSAIDs can be given to patients when the
benefits outweigh the risks. Local glucocorticoid injections have also been shown to be efficacious
but may carry an increased risk for plantar fascia rupture. Plantar fasciotomy is reserved for those
patients who have failed to improve after at least 6–12 months of conservative treatment.

You might also like